Contracts & Sales MBE Prep.

Pataasin ang iyong marka sa homework at exams ngayon gamit ang Quizwiz!

On January 8, a liquor company sent a signed, written offer to a retailer containing the following: "We will sell you our last 500 cases of our caffeine-infused vodka for $100 per case." Upon receiving the liquor company's offer, the retailer decided to take a couple of days to contemplate the offer. On January 15, after not hearing from the retailer, the liquor company decided it was no longer willing to sell the vodka to the retailer, but it did not contact the retailer. On January 17, the retailer decided that it was not interested in the vodka and placed a letter rejecting the offer in the mail. However, after hearing about the popularity of the vodka from a number of patrons that same day, the retailer immediately mailed a signed acceptance to the liquor company. The liquor company received the retailer's acceptance on January 20. Later that same day but unbeknownst to either party, the state enacted a law prohibiting the sale or distribution of the caffeine-infused vodka due to a number of serious health risks connected to the product. Due to an error at the post office, the retailer's initial rejection was not received until January 21. The retailer and the liquor company both learned of the state law on January 22. Which of the following is an accurate statement regarding the relationship between the retailer and the liquor company as of January 23? A. A contract was formed on January 20, but the duty to perform under the contract is discharged due to illegality. B. An enforceable contract was formed on January 20. C. No contract was formed because the liquor company's offer was revoked on January 15. D. No contract was formed because the retailer's rejection was effective on January 17.

A. A contract was formed on January 20, but the duty to perform under the contract is discharged due to illegality. *An offer can be revoked any time prior to acceptance, but a revocation is not effective until it is communicated to the offeree. If an offer is not revoked and the offeree mails a rejection before an acceptance, then the first communication received by the offeror controls.*

A restaurant placed an order for 100 pounds of USDA prime beef from a meat packer. The order was to be shipped to the restaurant immediately. The meat packer erroneously shipped 100 pounds of USDA choice beef, rather than prime beef. Which of the following best states the restaurant's rights and duties upon delivery of the beef? A. A contract was formed when the meat packer shipped the choice beef, but the restaurant is not required to accept the choice beef. B. A contract was formed when the meat packer shipped the choice beef, so the restaurant cannot reject the choice beef. C. No contract was formed because the meat packer's shipment of choice beef did not mirror the terms of the offer, so the restaurant is not required to accept the choice beef. D. The meat packer's shipment of choice beef constituted a counteroffer, and the restaurant can accept or reject the counteroffer.

A. A contract was formed when the meat packer shipped the choice beef, but the restaurant is not required to accept the choice beef. *Under the UCC, a buyer's request for shipment of goods is construed as an offer that the seller may accept by (1) promising to ship the goods, (2) shipping conforming goods, or (3) shipping nonconforming goods without notice of accommodation. Shipping nonconforming goods without such notice constitutes an immediate breach.*

At the auction of construction equipment owned by a contractor, several lots were offered for bidding and the highest bids for each were accepted by the auctioneer. The auctioneer then announced that a lot that consisted of a backhoe was being auctioned off. Several bids for the backhoe were acknowledged by the auctioneer. Just before the auctioneer brought down her gavel, she glanced at the contractor. The contractor gave the auctioneer a prearranged signal. Acting in accord with the signal, the auctioneer stated that the backhoe was being removed from the auction. There had been no indication as to whether the auction was being held with or without reserve. The highest bidder on the backhoe, contending that he is now its owner, has brought suit against the contractor. How is the court likely to rule? A. For the contractor, because the auctioneer had not brought down the gavel, announcing the completion of the sale of the backhoe. B. For the contractor, because the backhoe constituted equipment. C. For the highest bidder, because the contractor forfeited his right to withdraw the backhoe by prearranging a signal with the auctioneer. D. For the highest bidder, because the contractor lost the right to withdraw the backhoe once the auction began.

A. For the contractor, because the auctioneer had not brought down the gavel, announcing the completion of the sale of the backhoe. *The UCC has special rules for goods sold at auction. If goods are auctioned in lots, each lot represents a separate sale. Whether the goods can be withdrawn once the auctioneer calls for bids depends on the type of auction: 1) at a reserve auction -- which is presumed unless a no-reserve action is announced -- the auctioneer may withdraw goods from auction prior to completion of the sale. 2) at a no-reserve auction -- which must be specifically announced -- goods cannot be withdrawn from auction after the auctioneer calls for bids unless no bid is received with a reasonable time.* **During a reserve auction, the auctioneer may withdraw goods from auction prior to completion of the sale (e.g., before the auctioneer's hammer falls). At a no-reserve auction, goods generally cannot be withdrawn after the auctioneer calls for bids.**

A woman emailed her friend, stating that someday, she would like to buy the friend's teacup collection. The email stated, "When times aren't so tough, I would gladly pay $1,000 for them." The friend responded with an email stating, "That would be fine with me. I'd love for you to have them." The women did not exchange money or the teacups and did not see each other until a year later. When they did see each other, the friend apologized for forgetting about their discussion and told the woman she would deliver the teacups the next weekend and would accept a check at that time. The woman said that she did not remember the discussion but would pay $750 for the teacups. The friend responded, "Haven't we already discussed this? Sold." The next day, the friend turned the teacups over to the woman, who provided the friend with a check for $750. The friend immediately responded that she needed the check for the remaining $250. The woman kept the teacups. Is the woman liable for the remaining $250? A. No, because a contract was not formed until the day the women spoke in person. B. No, because oral agreements for the sale of goods are not enforceable. C. Yes, because the original contract was for $1,000. D. Yes, because the woman kept the teacups.

A. No, because a contract was not formed until the day the women spoke in person.

A supervisor and an employee in neighboring cubicles had been chatting about sports. Ten minutes after their sports discussion had ended, the supervisor offered to sell the employee tickets to an upcoming baseball game for $500, even though he saw the employee had placed noise-cancelling earphones into his ears and begun working again. The employee realized that he missed something that his supervisor had said and nervously responded, "Sure, boss." The supervisor honestly but unreasonably believed the employee had understood and accepted the offer. The day of the game, the supervisor told the employee that he would accept cash or check. When the employee indicated he had no idea what the supervisor was referencing, the supervisor angrily restated the terms of the discussion. Upon hearing the facts, the employee, who was short on funds, refused to pay the supervisor. Has a contract between the supervisor and employee been formed? A. No, because a reasonable person would not believe the parties had entered into a contract. B. No, because the employee did not honestly intend to accept the offer. C. Yes, because the employee accepted the terms of the offer. D. Yes, because the supervisor honestly believed the employee accepted the offer.

A. No, because a reasonable person would not believe the parties had entered into a contract. *The validity of an offer or an acceptance is determined by the objective theory of contracts, under which a party's intent is determined by what a reasonable person in the other party's position would believe considering the circumstances. A party's subjective intent is irrelevant.*

A car salesman on the lot at a car dealership specializing in expensive, high-end vehicles called out with a megaphone, "Free set of wheels to the next person who buys a car from me!" He was surrounded by stacks of tires and a display case with informational pamphlets advertising all-weather tires. A customer believed that the car salesman was offering a two-for-one deal on the expensive vehicles sold on the lot. Therefore, the customer immediately approached the salesman and purchased a new vehicle for his son, believing that he would have a nice new car to give to his daughter as well. The salesman then asked if the customer would like his free set of tires installed on the new car. The customer responded that he did not want the tires and that the salesman was contractually obligated to give him a second car. The salesman laughed and said that he was never offering a free car but would cancel the sale of the first car as a consolation. Is the salesman obligated to give the customer a free car? A. No, because no reasonable person would believe that the salesman was offering a free car. B. No, because the salesman was willing to cancel the sale. C. Yes, because only the customer has the right to avoid or reform the contract for misrepresentation. D. Yes, because the customer honestly believed that the salesman was making a valid offer.

A. No, because no reasonable person would believe that the salesman was offering a free car. *Under the objective theory of contracts, a party's intent to contract is judged by outward objective facts as interpreted by a reasonable person—not a party's subjective intent or belief.*

On May 10, the coach of a youth-league baseball team sent a letter to a supplier asking the supplier to promptly ship 20 red jerseys to the coach. On May 15, the supplier received this letter and sent the coach a reply letter accepting the offer. On May 16, the supplier realized that he had no red jerseys with which to fill the order and sent the coach 20 blue jerseys with a note that the blue jerseys were tendered as an accommodation. The coach received the jerseys and accommodation note on May 18 and received the supplier's acceptance letter on May 19. On May 20, which of the following is a correct statement of the parties' legal rights and duties? A. The coach can either accept or reject the blue jerseys and, in either event, recover damages, if any, for breach of contract. B. The coach can either accept or reject the blue jerseys, but if he rejects them, he will thereby waive any remedy for breach of contract. C. The supplier's shipment of nonconforming goods constituted an acceptance of the coach's offer, thereby creating a contract for the sale of the blue jerseys. D. The supplier's shipment of the blue jerseys constituted a counteroffer.

A. The coach can either accept or reject the blue jerseys and, in either event, recover damages, if any, for breach of contract. *Mailbox-rule: acceptance counts when the letter is sent. (only when acceptance is first; here it was first)

A comic-book collector agreed to purchase comic books from a man who had just inherited a large collection of them. Before they signed the written agreement, the collector emailed the man to ask if he would include a particularly rare issue of Batman in the sale, and the man agreed by email that he would. The written contract provided that the man would sell to the collector "a collection of comic books containing the following issues," followed by a list of each comic book to be included in the sale, for a total price of $4,000. The list did not include the Batman issue. The contract also stated that it was "the complete and final agreement" between the man and the collector. When the collector received the comic books, he discovered that the shipment did not include the Batman issue. He sued the man, stating that even though the list in the written contract did not mention the particular Batman issue, the collector had relied on the promise in the man's email when he signed the contract. At trial, the collector seeks to introduce the emails sent prior to the execution of the agreement that referenced the Batman issue. Are the emails admissible? A. No, because of the parol evidence rule. B. No, because the Uniform Commercial Code does not apply to this transaction. C. Yes, because the agreement was only partially integrated. D. Yes, because the parol evidence rule applies only to oral communications.

A. No, because of the parol evidence rule. *The UCC, which applies to contracts for the sale of goods (e.g., comic books), presumes that a contract is partially integrated. However, that presumption goes away when the writing contains a merger clause—i.e., a clause that declares the written contract to be the complete and final agreement between the parties. The written contract will instead be deemed completely integrated.* **And since parol evidence is inadmissible to supplement or contradict the written terms of a fully integrated agreement, the emails sent prior to the execution of the written agreement are not admissible.**

A gardener and a carpenter contracted in writing for the carpenter to repair the gardener's four identical beehives for $500 each. The contract was signed by both parties and provided that the gardener would pay the carpenter $2,000 upon delivery of the fourth repaired beehive. The gardener immediately delivered all four beehives to the carpenter for repair. The carpenter repaired and delivered the first two beehives without any problems and without demanding payment. However, upon delivery of the third repaired beehive, the carpenter demanded a payment of $1,500. Is the gardener required to make the demanded payment at this time? A. No, because she has no duty to pay the carpenter anything until the last beehive is repaired and delivered. B. No, because the course of performance between the parties has established that payment is not due upon the delivery of each repaired beehive. C. Yes, because the carpenter is entitled to the fair value conferred on the gardener. D. Yes, because the contract is divisible with respect to the repair and delivery of each beehive.

A. No, because she has no duty to pay the carpenter anything until the last beehive is repaired and delivered. *Although courts prefer to interpret contracts as divisible, they will not do so in contradiction of the contract's express terms—e.g., when the contract expressly states that it is indivisible.* **specifies that garden will pay carpenter at "delivery of the fourth repaired beehives**

The owner of a coffee shop saw the work of an eccentric local artist at an art show. The owner discovered that the artist operated a small interior-decorating business and, wanting the artist's unique style reflected in her own business, hired the artist to decorate her coffee shop. A week before the artist was scheduled to decorate the coffee shop, the artist sold her decorating business to a young art school graduate and delegated all of her outstanding contracts to him. The graduate took over all financial and creative management of the business. If the coffee shop owner refuses to accept performance by the art school graduate, is the owner liable for breach of contract? A. No, because the artist's duty under the contract involved her taste and skill. B. No, because the delegation created reasonable grounds for insecurity. C. Yes, because the art school graduate is completely capable of performing the contract. D. Yes, because the contract did not prohibit delegation of duty.

A. No, because the artist's duty under the contract involved her taste and skill. *Delegation of contractual duties is not permitted when (1) the other party to the contract has a substantial interest in having the delegating party perform or (2) the contract prohibits delegation.*

A bank that held a security interest in a delivery van conducted a forced sale of the van at an auction after the owner of the van defaulted on his loan from the bank. The proceeds of the loan had been used to purchase the van, which the owner had used in his floral business. At the auction, which was held in accordance with statutory requirements, the owner made a good-faith bid on the van but did not disclose his ownership interest. Twelve days after the auction, the highest bidder filed an action to void the sale after learning that one of the bidders had been the owner of the van. Is the highest bidder likely to succeed? A. No, because the auction was a forced sale of the van. B. No, because the highest bidder did not file his action to void the sale within 10 days of the auction. C. Yes, because the owner bid at the auction without disclosing his interest in the van. D. Yes, because the owner was a merchant.

A. No, because the auction was a forced sale of the van. *The UCC, which governs contracts for the sale of goods, provides special rules for auction sales. One such rule limits the seller's ability to bid at an auction sale. It allows a winning bidder to avoid the sale or pay the price of the last good-faith bid if the auctioneer (1) knowingly accepted a bid by the seller or on the seller's behalf or (2) procured the seller's bid to drive up the price of the goods. However, the winning bidder may not do so if: A. the seller bid at a forced sale (e.g., a foreclosure sale initiated by a secured creditor) OR B. the seller gave notice reserving the right to bid. **Here, the bank forced the owner to sell his van at an auction after the owner defaulted on his loan from the bank. Although the owner bid on the van at the auction sale, he was allowed to do so—even without disclosing his interest in the van or reserving the right to bid—because this was a forced sale.** ***A winning bidder may avoid an auction sale or pay the price of the last good-faith bid if the auctioneer (1) knowingly accepted a bid by the seller or on the seller's behalf or (2) procured the seller's bid to drive up the price. However, the winning bidder may not do so if the seller bid at a forced sale or gave notice reserving the right to bid.***

A caterer contracted with a local farmer for the delivery of three dozen fresh local eggs. The contract provided that because the caterer planned to use the eggshells to serve one of her signature dessert recipes, the eggs needed to be a uniform color. The farmer delivered the caterer 20 white eggs and 16 speckled eggs. The caterer immediately emailed the farmer and informed him that she was rejecting the eggs because she could not use the inconsistent shells to serve her desserts. The caterer also told the farmer that she did not have the ability to refrigerate the eggs or the space to store them for long and that she would wait for his instructions. The caterer stored the eggs on her countertop for a week and had not heard from the farmer. Concerned that the unrefrigerated eggs would soon spoil, the caterer promptly returned the eggs to the farmer. Due to the perishable nature of the eggs, the farmer had to resell the eggs at half the normal price. If the farmer brings a breach-of-contract claim against the caterer to recover the full contract price of the eggs, will he succeed? A. No, because the caterer behaved appropriately after rightfully rejecting the eggs. B. No, because the caterer had no obligations regarding the nonconforming eggs. C. Yes, because the caterer had a duty to retain the eggs until the farmer retrieved them. D. Yes, because the caterer was required to sell the eggs on the farmer's behalf.

A. No, because the caterer behaved appropriately after rightfully rejecting the eggs. *A buyer must retain rejected goods for a reasonable time to allow the seller to reclaim them. In the absence of other instructions, the buyer must sell the goods on the seller's behalf if the buyer is a merchant, the goods are perishable, and there is no local agent to whom the goods can be returned.*

During the warm months of the year, the owner of a fur coat stored it with the furrier from whom she had bought it. While the coat was at the furrier's store, a salesperson, mistakenly thinking that the coat was for sale, sold it to a customer. The customer was allowed to reduce the purchase price by the amount of an outstanding debt owed by the furrier to the customer; the customer paid the remainder in cash. In the process of purchasing the coat, the customer was told by the salesperson about the furrier's storage service but, like the salesperson, was unaware that the coat was not part of the store's merchandise. After the sale, the owner learned of the transaction between the furrier and the customer. Since the coat had significant sentimental value to the owner, she sought its return from the customer. When the customer refused, the owner filed an action to recover the coat from the customer. Will the owner likely prevail? A. No, because the customer was a good-faith purchaser of the coat that had been entrusted to the furrier. B. No, because the owner is entitled to damages from the furrier. C. Yes, because the customer did not give full value in acquiring the coat. D. Yes, because the furrier transferred only voidable title in the coat to the customer.

A. No, because the customer was a good-faith purchaser of the coat that had been entrusted to the furrier. *A merchant entrusted with goods has the power to convey good title to a buyer in the ordinary course. A buyer in the ordinary course is someone who buys goods (1) in good faith, (2) without knowledge that the sale violates the owner's rights to the goods, and (3) from a merchant in the business of selling goods of that kind.*

A wheat farmer contacted an agricultural services company in May to inquire about hiring workers for a five-day period toward the beginning of the summer-long harvest season to assist the farmer in harvesting his wheat crop. After some negotiations, the farmer entered into a written contract with the company "to provide five workers for a five-day period starting in the first week of June for a cost of $5,000." On May 31, the company's workers went on strike. On June 9, the strike ended, and the company's workers began harvesting wheat on the farmer's farm for the next five days. The farmer subsequently refused to pay the company, claiming that the company's delay in performance excused his obligation to pay. Is the farmer's obligation to pay excused? A. No, because the delay did not deprive the farmer of the substantial benefit of the bargain. B. Yes, because starting in the first week of June was an express condition of the contract. C. Yes, because substantial performance does not excuse a breach for commercial contracts. D. Yes, because the delay was a material breach as the harvesting season had already begun.

A. No, because the delay did not deprive the farmer of the substantial benefit of the bargain. *Under common law, a material breach of contract allows the nonbreaching party to withhold performance. A breach is material when the nonbreaching party does not receive the substantial benefit of its bargain, so substantial performance does not constitute a material breach (except as to an express condition).* *There was no express condition for work to start the first week of June. Look for "on the condition that" or "provided that."*

The owner of a ferry boat operated the boat only during daylight hours during the summer months of June, July, and August. On March 1, the owner entered into a written agreement with a man to serve as the captain of the boat for the upcoming season. On May 1, the owner contracted with a woman to serve as the captain of the boat. On May 30, the man was diagnosed with an illness, and the treatment for this illness prevented him from being employed until the following year. On May 31, the owner learned of the man's illness and told the man not to worry about their contract as he had found someone else to serve as captain of the boat. The woman served as captain of the boat for the summer months of June, July, and August that year. On September 1, the man sued the owner for damages based on a breach of their contract. Can the man recover damages based on breach of contract? A. No, because the man was unable to serve as the captain of the boat during the summer months. B. No, because the owner informed the man about the owner's contract with the woman prior to June 1. C. Yes, because the owner did not inform the man of the owner's contract with the woman until after the owner learned of the man's illness. D. Yes, because the owner's contract with the woman constituted an anticipatory breach of the owner's contract with the man.

A. No, because the man was unable to serve as the captain of the boat during the summer months. *A nonrepudiating party who materially breaches the contract cannot recover damages for the other party's anticipatory breach because the material breach discharges the other party's duty to perform.*

A men's apparel wholesaler was trying to expand its business, so it reached out to an online company that sold men's ties. The wholesaler mailed a letter to the online company offering to sell it 1,000 silk ties at a wholesale price of $15 per tie. The signed letter, dated July 1, assured the online company that the option to purchase would stay open but did not specifically state how long the option would remain open. A year later, the online company sent a letter to the wholesaler accepting its offer to sell the company 1,000 silk ties at a wholesale price of $15 per tie. The wholesaler and online company had no prior dealings, and offers of this kind in the industry generally do not remain open for a year with no further contact between the parties. If no other correspondence or action was taken by either party between the wholesaler's offer and the online company's purported acceptance, is there an enforceable contract between the parties? A. No, because the online company did not accept the offer within a reasonable period of time. B. No, because the online company did not give consideration to keep the offer open. C. Yes, because the time period of irrevocability was not stated. D. Yes, because the wholesaler's offer had no termination date.

A. No, because the online company did not accept the offer within a reasonable period of time. *An offer will terminate by lapse after a reasonable period of time if no termination date is specified. Reasonableness is determined by several factors, including (1) the nature of the contract, (2) the purpose and course of dealing between the parties, and (3) trade usage.*

The owner of a rare eighteenth-century chest offered to sell it to a connoisseur of antiques for $75,000. The connoisseur countered that she would buy the chest for $50,000. The owner rejected this price. The owner and the connoisseur then executed a written agreement for the sale of the chest at a price to be determined only by a particular antiques dealer whose expertise in valuing this rare item they both trusted. Two weeks later, the agreed-upon antiques dealer examined the chest. He told the owner and the connoisseur that he had to do further research on the chest but that he would let them know his decision in several days. Unfortunately, the dealer died before doing so. A reasonable price for the chest can be established by the court. Is there likely an enforceable contract? A. No, because the owner and the connoisseur did not intend to be bound unless the dealer set the price of the chest. B. No, because the price of the chest was not determined at the time the agreement was executed. C. Yes, because a reasonable price for the chest can b established by the court. D. Yes, because the owner and the connoisseur executed a written agreement for the sale of the chest.

A. No, because the owner and the connoisseur did not intend to be bound unless the dealer set the price of the chest. *Under the UCC, a contract for the sale of goods is formed if both parties intend to contract and there is a reasonably certain basis for giving a remedy in the event of a breach. -> both parties did not intend to contract until the price was set. (objective manifestation of intent) -> no contract.

The owner of a beauty products store mentioned to a longtime customer that she was selling her car. The storeowner showed the customer a picture of the car and told her its year, make, model, and mileage. When the customer expressed an interest, the storeowner gave her the keys and told her to check it out for herself. The customer took the keys, looked over the inside and the outside of the car, and drove it around the block. When the customer returned to the store, the storeowner honestly stated that she knew little about cars and was selling the car with all its faults. The storeowner and the customer agreed upon a price of several thousand dollars for the car. Several days after the customer paid for the car and took ownership of it, the car stopped running. The customer towed the car to a mechanic and learned that it required a costly engine overhaul that neither the storeowner nor the customer was aware of at the time of the sale and that could not have been detected without a specialized inspection. The customer has filed a lawsuit against the storeowner for breach of the warranty of merchantability. Is the customer likely to be successful? A. No, because the storeowner was not a merchant with respect to the car. B. No, because the storeowner was unaware of the problem with the car's engine. C. Yes, because the defect could not have been detected without a specialized inspection. D. Yes, because the warranty of merchantability cannot be orally disclaimed.

A. No, because the storeowner was not a merchant with respect to the car.

While attending a rodeo on August 20, a hatmaker entered into a valid, written agreement with the rodeo manager to make 500 leather cowboy hats for an upcoming rodeo event at a price of $75 per hat. Per the agreement, the rodeo manager agreed to pay one-fourth of the total purchase price to a tannery owner to whom the hatmaker owed a debt for a previous leather order. The hatmaker and the rodeo manager made no mention of the agreement to the tannery owner. On August 25, the hatmaker changed his mind about paying one-fourth of the purchase price to the tannery owner. The hatmaker and the rodeo manager subsequently executed a valid modification of the original agreement. The rodeo manager's brother had also been present on August 20 when the original agreement was executed, but he did not know about the August 25 modification of the agreement to no longer pay the tannery owner. On August 30, the brother, who was friends with the tannery owner, called and told the tannery owner that his debt from the hatmaker would finally be paid off. However, the rodeo manager refused to pay one-fourth of the purchase price to the tannery owner. If the tannery owner sues the rodeo manager for one-fourth of the purchase price, will he likely recover? A. No, because the tannery owner did not rely on the August 20 agreement between the hatmaker and the rodeo manager. B. No, because there was no consideration for the promise to pay the tannery owner by the hatmaker and the rodeo manager. C. Yes, because the tannery owner had the right to sue the rodeo manager to enforce the contract between the rodeo manager and the hatmaker. D. Yes, because the rodeo manager agreed to pay one-fourth of the purchase price to the tannery owner on August 20.

A. No, because the tannery owner did not rely on the August 20 agreement between the hatmaker and the rodeo manager. *Intended beneficiary: Can recover is rights vested through: 1) detrimental reliance; 2) manifestation of assent; OR 3) lawsuit *While Tannery owner was an intended benefiary, the contract was modified BEFORE Tennery owner learned about the original agreement, therefore, his right to recover never vested.*

A builder ordered 100 squares of shingles from a home-supply store for installation on the roofs of homes that he was building. The builder agreed to a price of $120 per square. Delivery to the construction site was set for no later than noon on the following Monday. The store's truck with the ordered shingles arrived at 1:00 p.m. the following Monday. The builder rejected the shipment due to its failure to arrive on time. The store, which regularly sold 600 squares of shingles per week, resold the squares that had been rejected by the builder at a price of $110 per square. The store would have made a profit of $3,000 had the builder accepted the shingles. If the store sues the builder for breach of contract, how much can the store recover from the builder? A. Nothing B. $1,000, the contract price minus the resale price. C. $3,000, the store's lost profit on the initial sale. D. $4,000, to recover the store's total expectation damages.

A. Nothing *The UCC requires perfect tender. If the buyer rejects goods for imperfect tender and the seller is unable to cure, then the seller is in breach and cannot recover damages under the contract.*

A retailer received a written firm offer signed by a supplier. The offer committed the supplier to providing the retailer with up to 10,000 tubes of toothpaste over the next 45 days at $1.00 a tube. Thirty days later, the supplier informed the retailer that the price per tube of toothpaste would be $1.10. The next day, the retailer ordered 6,000 tubes of toothpaste from the supplier, which the supplier promptly shipped. Sixty days after the receipt of the offer, the retailer ordered another 4,000 tubes of toothpaste, which the supplier also promptly shipped. What price is the supplier permitted to charge the retailer for the toothpaste? A. $10,000 (10,000 × $1.00), because the supplier's firm offer was effective for three months regardless of its terms. B. $10,400 ((6,000 × $1.00) + (4,000 × $1.10)), because the supplier's firm offer was effective for only 45 days. C. $11,000 (10,000 x $1.10), because the firm offer rule does not apply where the buyer is merchant. D. $11,000 (10,000 x $1.10), because the supplier informed the retailer that the price was increased to $1.10 before the retailer placed either order.

B. $10,400 ((6,000 × $1.00) + (4,000 × $1.10)), because the supplier's firm offer was effective for only 45 days. *Firm Offer rule applies to only the 6,000 tubes of toothpaste (that was within the 45 day period). The 4,000 were ordered outside of the 45 day term*

A wholesaler of bicycle chains sent a retailer the following fax on December 1: "Because of your continued loyalty as a customer, I am prepared to sell you up to 1,000 units of Bicycle Chain Model D at $7.50 per unit, a 25% discount off our original $10.00 price. This offer will remain open for 7 days." The fax lacked a full, handwritten signature, but it was on the wholesaler's letterhead and had been initialed by the wholesaler's head of sales. On December 4, the wholesaler's head of sales called the retailer and informed the retailer that the wholesaler had decided to revoke his December 1 offer. On December 5, the retailer placed an order for 1,000 bicycle chains, stating that he would pay the discounted price of $7.50 per unit. What is the correct value of the order placed by the retailer? A. $7,500, because the wholesaler's revocation was not in writing. B. $7,500, because the wholesaler was bound to keep the offer open for 7 days. C. $10,000, because the offer was not signed by the wholesaler. D. $10,000, because the retailer did not provide consideration to hold the offer open.

B. $7,500, because the wholesaler was bound to keep the offer open for 7 days. *Firm Offer (UCC) rule* 1) seller be merchant; 2) written promise to to revoke; 3) signature of seller (offeror) [initial counts!]

At the beginning of the week, a homeowner met with a contractor to discuss remodeling a bathroom in her home. At the conclusion of their meeting, the contractor told the homeowner that he would charge her $5,000 to $6,000 for the work, but that he would get back to her with a final price. When he arrived at his office later that day, the contractor opened an email from the homeowner that she had sent earlier. In the email, she stated that she would pay the contractor $5,000 for the job. Two days later, the contractor responded by email that he could not complete the work for less than $5,500. The homeowner replied by email that she couldn't pay $5,500 but that, if the contractor changed his mind about doing the work for $5,000, he could begin work before the end of the week. The contractor received the email but did not respond. The next day, the contractor appeared at the homeowner's house and began remodeling the bathroom. Which of the following statements regarding the relationship between the parties is most accurate? A. A contract was formed at a reasonable price. B. A contract was formed at the price of $5,000. C. A contract was formed at the price of $5,500. D. No contract was formed.

B. A contract was formed at the price of $5,000. *An offer cannot be accepted after it terminates (e.g., is rejected by the offeree). But the offer can be revived if the offeror conveys that it is still open, which creates a renewed opportunity for the offeree to accept.*

A jeweler who specialized in engagement rings assisted a man who was trying to pick out the perfect engagement ring. The man was inexperienced with the various cuts of diamonds and types of ring settings. Over the course of a few weeks, the jeweler and the man looked at all of the ring styles and discussed pricing based on the man's budget of $5,000. The man finally settled upon a square-cut diamond with a prong setting that was priced at $5,500. The man initially offered the jeweler $4,500 for the ring. While the man and the jeweler were negotiating the price, the jeweler received a phone call regarding a family emergency. The jeweler told the man that he would email him an offer in the evening, and if they could "meet halfway," the jeweler would sell the ring to the man. The man agreed. That evening, the jeweler and the man received emails from one another at the same time. The jeweler's email contained an offer to sell the ring for $5,000, and the man's email contained an offer to buy the ring for $5,000. Both emails (i) specified the same style of ring that the two parties had discussed earlier that day, (ii) required payment upon receipt of the ring in two weeks, and (iii) were signed with an electronic signature. Based upon their earlier discussions and the jeweler's email offer to sell the ring to him for $5,000, the man did not look for an engagement ring at any other jewelry store. When the man showed up two weeks later to pick up and pay for the ring, the jeweler denied that they had a binding contract and would not sell the ring. If the man sues the jeweler for breach of contract, which of the following most persuasively supports the man's position? A. A sale-of-goods contract does not require that an acceptance be a mirror image of the offer. B. Both parties conveyed an intent to contract with one another through prior negotiations and the simultaneous emails. C. Since the jeweler was the only merchant in the transaction, the jeweler is estopped from denying that the parties' correspondence created a binding contract. D. The man detrimentally relied upon the jeweler's offer to "meet halfway" and the email offer to sell the ring to him.

B. Both parties conveyed an intent to contract with one another through prior negotiations and the simultaneous emails. *Under the UCC, a contract is formed if the parties intended to contract and there is a reasonably certain basis for giving a remedy—even if the moment of formation is uncertain.*

A general contractor was preparing a bid to build a "green home" designed to be environmentally friendly and sustainable. The general contractor received five bids from subcontractors ranging from $22,000 to $29,000 for the installation of solar panels on the roof of the house. In computing his own bid, the general contractor used the lowest sub-bid of $22,000 for installation of the solar panels. The general contractor was awarded the contract to build the green home. After winning the bid, the general contractor was approached by another solar panel installer, who offered to install the solar panels for $20,000. The general contractor entered into a contract with this installer to mount solar panels onto the roof of the green home. The subcontractor who had submitted the $22,000 sub-bid learned of the general contractor's contract with the installer and sued the general contractor for breach of contract. Will he succeed? A. No, because his sub bid was higher than the sub bid accepted by the general contractor. B. No, because his sub-bid was never accepted. C. Yes, because of detrimental reliance. D. Yes, because the general contractor was bound to accept his sub-bid.

B. No, because his sub-bid was never accepted. *When subcontractor bids, it functions as an outstanding offer. When general contractor submits a bid, the subcontractor's bid becomes IRREVOCABLE. *If general contract accepts subcontractor's bid = Contract is FORMED. *if general contract rejects subcontractor's bid = No Contract is formed.*

Upon the completion of an interview, an attorney offered an assistant $3,000 each month to perform administrative tasks for his law firm. The attorney emailed the assistant an offer confirming these terms. The assistant responded via email, stating that she accepted the offer so long as she could work remotely, expressing that the option to work from home was essential to her. The parties had discussed in the interview that the assistant would likely work in the office but that the attorney would be open to her working remotely. The emailed offer did not address this issue. The attorney did not respond that day but went out and bought the assistant a laptop conforming to her preferences and many office supplies that she had requested. The next day, the attorney emailed the assistant to confirm that he might allow her to work remotely after she demonstrated success within an office environment. She emailed the attorney back, stating that she no longer wished to work with him because she preferred to work from home right away. The attorney threatened to sue her for breach of contract. Does a contract exist that binds the attorney and the assistant? A. No, because the assistant had a misunderstanding as to the terms of the contract. B. No, because the assistant included additional terms in her response. C. Yes, because the assistant emailed him back to accept the offer. D. Yes, because the attorney's offer did not propose that the assistant could work from home

B. No, because the assistant included additional terms in her response. *Under the common-law mirror-image rule, an acceptance must match the terms of the offer exactly to be effective. Therefore, an acceptance acts as a counteroffer—i.e., a rejection of the original offer and a new offer—if it is conditioned upon the offeror's assent to different or additional terms.*

A baker and a bride-to-be entered into a contract in which the baker agreed to bake the wedding cake for the bride's wedding at a cost of $2,500. The contract contained a clause that read: "Bride's obligation to perform under the Contract is expressly conditioned upon her complete satisfaction with Baker's aesthetic design of her wedding cake." In keeping with the wedding's butterfly theme, the baker constructed an elegant cake accented with colorful butterflies, flowers, and caterpillars. At the wedding reception, the guests were enthralled by the cake. However, the bride had a genuine aversion to caterpillars, and she hated the cake. When the bride refused to pay the baker, he sued her for $2,500. Should the court find that the bride has breached the contract? A. No, because no contract was formed between the parties. B. No, because the bride was personally and honestly dissatisfied with the cake. C. Yes, because the baker substantially performed. D. Yes, because the cake was aesthetically pleasing to the wedding guests.

B. No, because the bride was personally and honestly dissatisfied with the cake. *Satisfaction contract: generally contract law is dictated by the "Reasonable Person" standard (objective). However, a satisfaction contract based in the individual aesthetic taste of an individual relies on their Subjective standard. Reasonable person doesn't apply.

On April 1, a buyer agreed in writing to purchase an antique car from a seller for $20,000. The parties met on April 10, the scheduled date of the sale, at which time the buyer accepted the car and gave the seller a check for $15,000. The buyer, seeking to create an accord and satisfaction, had added the following conspicuous notation on the check: "This check is in full and final satisfaction of my obligation under our April 1 agreement." The seller did not realize that the check was for only $15,000 and that it contained the notation until the seller sought to deposit it at her bank later that day. Needing the money, the seller deposited the check anyway. If the seller sues the buyer for breach of contract seeking damages of $5,000, the difference between the amount paid and the contract price, will the buyer's accord and satisfaction defense likely succeed? A. No, because the buyer could not modify the agreement without consideration. B. No, because the buyer did not dispute the initial purchase price of the car. C. Yes, because the notation on the check formed a substituted contract. D. Yes, because the seller deposited the check knowing it was offered in full and final satisfaction of the buyer's obligation.

B. No, because the buyer did not dispute the initial purchase price of the car. *Under an accord and satisfaction, a party can fulfill its contractual obligation by rendering different performance than the one initially promised. This can be accomplished through a negotiable instrument (e.g., check) if three conditions are met: 1) The obligation is unliquidatee (i.e., uncertain in amount) or otherwise in dispute; 2) The obligor, in good faith, tenders the negotiable instrument with a conspicuous statement that the instrument is tendered as full satisfaction of the obligation AND 3) The obligee obtains payment of the instrument (e.g., by cashing the check DISCHARGE)* **There was no dispute of the initial price. Therefore, no accord.**

Prior to her death, a celebrity commissioned an artist to paint a portrait of her. The celebrity hired this particular artist because he painted using an old-fashioned and rarely used style that required two months of daily appointments during which the subject would sit for a few hours each day. The contract between the parties specified that this live-model method would be used and that the celebrity would deliver increasing payments throughout the process, with the first payment occurring after two weeks of painting. One week into the process, after the painting had begun, the celebrity died. Her family demanded that the artist continue with the painting, using photographs as a substitute for the daily sessions. Is the artist required to complete a painting of the celebrity? A. No, because no payment had yet occurred. B. No, because the celebrity died after only one week. C. Yes, because the artist can complete the painting by relying on photos of the celebrity. D. Yes, because the artist had already begun painting the celebrity.

B. No, because the celebrity died after only one week. *In a personal-services contract (e.g., contract to paint a portrait), impracticability can arise when the party who is to perform the contract—or a person whose existence is required for that performance—dies or becomes incapacitated.* **A contracting party's duty to perform is discharged by impracticability when (1) an unanticipated or extraordinary event makes it impracticable for the party to perform, (2) the contract was formed under a basic assumption that the event would not occur, and (3) the party seeking discharge was not at fault in causing the event to occur.**

A chemistry professor offered to sell her colleague an autographed first edition of a novel for $1,000. The professor provided her colleague with a signed written statement specifying the terms of the offer and stating that the offer would remain open for one week. Two days later, the colleague learned that the professor had sold the book to someone else in their department. The next day, the colleague showed up at the professor's office with $1,000, asking to purchase the book. The professor apologized, saying that the book had already been sold. Is the colleague likely to succeed in an action for breach of contract? A. No, because an option contract is not valid unless the offeror is a merchant. B. No, because the colleague learned that the book had been sold before accepting the offer. C. Yes, because the professor did not revoke the offer prior to the colleague's acceptance. D. Yes, because the offer was contained in a signed writing and thus could not be revoked.

B. No, because the colleague learned that the book had been sold before accepting the offer. *If an offeree acquires reliable information that the offeror has taken definite action inconsistent with the offer, the offer is automatically revoked (i.e., constructive revocation) and can no longer be accepted. *also, option contracts do not requires parties to be merchants; the professor was not a merchant so their written statement wasn't a firm offer either.*

As part of a divorce settlement, an ex-husband purchased an annuity from an insurance company to be paid to his ex-wife so that she would receive a fixed amount quarterly for the duration of her life. Within a week after the purchase, the ex-wife learned that she had a fatal illness, which had not previously manifested itself but had existed for some time. She died two months later, prior to receiving any payments from the annuity. The ex-husband has filed suit to rescind the annuity contract. Will the ex-husband be likely to prevail? A. No, because the annuity contract was a third-party beneficiary contract. B. No, because the ex husband assumed the risk of his ex-wife's death. C. Yes, because the ex-wife's death frustrated the purpose of the annuity. D. Yes, because the ex-husband and the insurance company made a mutual mistake as to the ex-wife's heath.

B. No, because the ex-husband assumed the risk of his ex-wife's death. *Mistake as grounds for rescission: Mutual Mistake vs. Unilateral Mistake Mutual mistake: 1. the mistake relates to a basic assumption of the contract; 2. the mistake materially affects the agreed-upon exchange of performances; AND 3. the adversely affected party did NOT assume the risk of mistake. Unilateral mistake: mutual mistake factors PLUS: 1. mistake makes enforcement of contract unconscionable OR 2. nonmistaken party caused, or knew or should have known of, mistake. *Party assumes the risk of mistake if, at the time the contract is formed, the party is aware that he/she has limited knowledge of the facts and accepts this knowledge as sufficient.* *Because an annuity contract for the duration of someone's life assumes that the person will die but does not predict WHEN the death will occur. As such, there is an inherent risk of death before the purchase price is recouped.* *Frustration of purpose doesn't apply because you cannot form an annuity with the assumption that death will NOT occur.*

A licensing agreement provided that a manufacturer could use an inventor's patent in manufacturing its products for 10 years. Immediately thereafter, the inventor assigned his rights to receive payments pursuant to the licensing agreement to a corporation. The inventor did not receive compensation for this assignment. The inventor, upon his death five years later, devised his stock in the corporation to his daughter and all of his remaining property to his son. To whom should the manufacturer make its payments under the licensing agreement? A. The corporation. B. The inventor's daughter. C. The inventor's son. D. No one, because the manufacturer's obligation to make payments under the licensing agreement terminated upon the death of the inventor.

B. The inventor's son. *Assignments of contractual rights are revocable UNLESS: 1. obligor already performed; 2. document symbolizing assigned right (eg. stock certificate) delivered; 3. written & signed assignment delivered; 4. promissory estoppel applies.* *Assignor warrants that he/she: 1. has right to assign; 2. is not subject to limitations/defenses unknown to assignee; 3. will not defeat/impair assigned rights* *Because there was no consideration the inventor's assignment of rights was gratuitous and therefore, revocable upon his death. Son inherited all property besides the stock, therefore son gets the payments.*

The owner of a restaurant who highlighted local ingredients when creating his menu bought cheese and other dairy products from a local dairy farmer. The owner and the farmer had entered into written requirements contracts each spring for the past 10 years. In the winter of the tenth year, the farmer purchased a substantial amount of new dairy cows and expanded his farming capabilities. He notified all customers that he would have a higher volume and amount of available products the following spring and would adjust deliveries accordingly. The owner responded with a date he wished the products to be delivered, as per custom, but said nothing else. On the agreed-upon date, the farmer delivered substantially more products than he had customarily provided. The owner attempted to accept half of the shipment, as that was roughly his customary quantity, but the farmer stated that the products were already packaged and that the owner should have spoken up after receiving the notice from the farmer. The owner then rejected the shipment in its entirety. Did the owner breach the contract with the farmer as to this shipment? A. No, because no contract existed, as the parties did not agree to a quantity. B. No, because the farmer made a nonconforming tender of goods. C. Yes, because the owner should have given the farmer time to cure the nonconformity. D. Yes, because the owner rejected the shipment in its entirety.

B. No, because the farmer made a nonconforming tender of goods. *Perfect-tender rule (Requirements Contract) Perfect-tender = Perfect Goods & Perfect Delivery *This was a requirements contract, meaning seller must provide buyer 100% of what they may require (no more than)*

A seller called a buyer on the phone and offered his antique sports car to the buyer for $10,000. The buyer, wanting time to consider the offer, paid the seller $50 in exchange for an option keeping the offer open for 48 hours. The morning after the parties discussed the offer on the phone, the seller offered to drive the car to the buyer's home so that the buyer could inspect the car, see how it looked in his own driveway, and make a final decision on whether he wanted to buy it. On the way to the buyer's house, the seller was in an accident that totaled the car through no fault of the seller. Before the seller called to tell the buyer about the accident, the buyer called him to accept the offer. Once he learned of the accident, the buyer demanded that the seller provide him with a similar antique sports car. Is the seller required to provide the buyer with a similar antique sports car? A. No, because the buyer did not accept the offer during the original phone call. B. No, because the first car was destroyed. C. Yes, because driving the first car to the buyer's home constituted partial performance. D. Yes, because the first car would not have been destroyed but for the drive to show it to the buyer.

B. No, because the first car was destroyed. *

A buyer at a local market offered to purchase a large mirror from an artist for $1,000. The artist stated that he wanted to wait to see how many people went through the market that day before he decided on whether he would accept the offer. The buyer agreed to wait until the next morning for the artist's decision. The next morning, the buyer returned to the market only to learn that the mirror had been dropped and shattered. The buyer believed that the destruction of the mirror terminated his original offer, but because the frame of the mirror was still in good condition, the buyer decided to buy the frame instead. The buyer wrote a check for $500 and gave it to the artist without further remark. The artist loaded the empty frame into the buyer's vehicle and, believing that he had accepted the buyer's original offer, demanded the remaining $500 the buyer had offered the day before. Is the buyer liable for the remaining $500? A. No, because the buyer believed that the original offer had terminated. B. No, because the original offer terminated. C. Yes, because the artist thought that he had accepted the original offer. D. Yes, because the original offer was still valid.

B. No, because the original offer terminated. *An offer can be terminated by operation of law—e.g., when the subject matter of the offer is destroyed.* *Here, the buyer originally offered to purchase a large mirroor for $1,000. The offer terminated once the mirror -- the subject matter of the offer -- shattered.* **remember, objective theory of contract.**

A shoe manufacturer contends that the owner of a shoe store called and ordered 50 pairs of Oxford-style dress shoes at $100 per pair to be shipped within three weeks. The manufacturer promptly sent the owner a signed, written acknowledgment of the alleged order that reflected the manufacturer as seller and the owner as buyer, as well as the number and style of shoes. However, the acknowledgment did not indicate the price of the shoes. The owner admits to receiving the acknowledgment the following day and taking no action regarding it. Two weeks later, the owner received a shipment of 50 pairs of Oxford-style dress shoes. The owner immediately called the manufacturer and asserted that he had never ordered the shoes. Will the statute of frauds prevent the manufacturer from enforcing this contract against the owner? A. No, because an oral contract between merchants is enforceable. B. No, because the owner received and did not respond to the written acknowledgment in a timely manner. C. Yes, because the acknowledgment did not indicate the price of the shoes. D. Yes, because the price of the shoes exceeds the $500 threshold of the statute of frauds.

B. No, because the owner received and did not respond to the written acknowledgment in a timely manner. *Statute of Frauds: UCC statute of frauds applies for goods valued at $500 or more. To be enforceable, these agreements must be evidenced by a writing that: 1. provides a reasonable basis to believe a contract was formed (the written acknowledgment) 2. identifies the parties (the manufacturer as seller and the owner as buyer) 3. lists the quantity of goods sold (50 pairs of shoes) and 4. is signed by the party against whom enforcement is sought (the owner) * -> note, this requirement is lax under the MERCHANT EXCEPTION: a written confirmation need only be signed by one merchant and sent to the other merchant.* *if the recipient has reason to know the confirmation's contents and does not object within 10 days, then it is enforceable against both merchants.

A math tutor entered into an agreement with a father to provide one month of tutoring for the father's son. The agreement stated that the math tutor would provide lessons for the son twice a week during the month for eight lessons at a total cost of $1,000. After signing the contract, the math tutor told the father that he planned to put the first $350 of the cost of the lessons toward purchasing study materials from a particular educational services provider. The father knew a salesman for the educational services provider and notified him that his company should expect a sale in the next week. As the educational services provider was a new company without many sales, the salesman was excited at the prospect. A week after the agreement between the math tutor and the father was executed, the math tutor informed the father that he would not tutor the son unless the father provided an additional $250. The father refused. The educational services provider subsequently brought an action against the math tutor for breach of the agreement between the math tutor and the father. Will the educational services provider likely prevail? A. No, because the provider and the father were not in privity of contract. B. No, because the provider was only an incidental beneficiary of the agreement. C. Yes, because the father informed the provider's salesman of the agreement concerning its materials. D. Yes, because the provider sustained $350 in lost-profit damages as a result of the math tutor's breach.

B. No, because the provider was only an incidental beneficiary of the agreement. *Intended beneficiaries receive a direct benefit from a contract because the contracting parties so intended, while incidental beneficiaries receive an indirect benefit from a contract even though there was no contractual intent to benefit them. Only intended (not incidental) beneficiaries can sue to enforce the contract.*

A math major at a local college agreed to tutor a high school student who was having trouble in his math class. The tutor promised to meet with the student for five hours a week for the remaining two months of the term, and the student's mother agreed to pay the tutor $20 per hour. The tutor and the mother also agreed that, if the student received a B or better in his math class, the mother would pay the tutor a $500 bonus. The parties did not reduce their agreement to writing. The mother paid the tutor weekly for the tutoring. At the end of the term, the student received a B+ in his math class. The mother, not having the funds to pay the tutor's bonus, contacted the tutor and offered her a bike worth $300 in lieu of the bonus payment. The tutor accepted the bike, which the mother delivered to the tutor shortly thereafter. Is the tutor entitled to recover $200 from the mother? A. No, because the bike was worth less than $500. B. No, because the tutor accepted the bike in lieu of the bonus. C. Yes, because the modification was not in writing. D. Yes, because there was no dispute as to the amount owed.

B. No, because the tutor accepted the bike in lieu of the bonus. *Ways to discharge contractual obligations: FIRM SCAN 1. Full performance of contractual obligations; 2. Impossibility, impracticability, or frustration of purpose; 3. Release (in writing only); 4. Mutual rescission; 5. Substituted contract; 6. Contract or covenant not to sue; 7. Accord & satisfaction 8. Novation* **Where the new consideration is worth less than what was agreed to in the original contract, it will be sufficient only if: A. There is a good-faith dispute as to the amount owed; OR B. The new consideration is of a different type than what was owed under the original contract (e.g., goods in lieu of cash). ***here, the bike serves as a different type of new consideration.***

A mechanic and a farmer contracted in writing for the repair of the farmer's tractor, with a payment of $2,000 due upon completion. The mechanic called the farmer on April 15 to inform him that the work was complete. When the farmer went to pick up the tractor the next day, he told the mechanic that due to an unforeseen rise in feed costs, he could not pay the full contract price. The farmer paid the mechanic $1,000. The mechanic told the farmer that, if the farmer promised to pay the remainder by June 1, then the mechanic would not sue to recover the remaining $1,000. The farmer orally agreed. On May 1, the mechanic sued the farmer for the unpaid $1,000, and the farmer filed a motion to dismiss. Should the court grant the motion to dismiss? A. No, because the new cost of food is not an unforeseen difficulty that would allow for modification of the existing contract. B. No, because there is no consideration to support the mechanic's promise not to sue. C. Yes, because a promise to allow a debtor to delay payment on a past debt is enforceable without consideration. D. Yes, because the payment of $1,000 was sufficient consideration to support the mechanic's promise not to sue.

B. No, because there is no consideration to support the mechanic's promise not to sue. *Under the preexisting-duty rule, a promise to perform a duty that a party is already legally bound to perform is not consideration.* **Farmer hired mechanic for services meaning this is under the Common law. Therefore, pre-existing duty rule applies.**

A homeowner entered into a contract with a landscaper. The contract specified that the homeowner would pay the landscaper $10,000 upon completion of a list of projects. The landscaper performed the work while the homeowner was away on vacation. When the landscaper sought payment, the homeowner refused, noting that a tree had not been trimmed as required by the contract. The landscaper responded that, since he would now have to forego other work in order to trim the tree, he would do it but only if the homeowner agreed to pay him a total of $10,500 for his services. The homeowner, desperate to have the work completed, agreed. Once the work was completed, however, the homeowner gave the landscaper a check for $10,000 and refused to pay more. The landscaper sued for breach of contract. Is the landscaper likely to succeed in his claim? A. No, because an enforceable contract cannot be renegotiated. B. No, because there was no consideration for the promise to pay $10,500 and no unanticipated circumstances arose. C. Yes, because there was a valid modification of the contract. D. Yes, because the landscaper suffered a detriment by foregoing other work.

B. No, because there was no consideration for the promise to pay $10,500 and no unanticipated circumstances arose. *Under Common Law, pre-existing duty rule applies.* *because landscaper already agreed to do the work in the previous contract -> it is not consideration to do what you are already legally contracted to do.*

The owner of a retail clothing store regularly displayed for-sale works by local artists on a wall in the store. An art collector who came into the store inquired about purchasing a particular work for display at his home. The two agreed upon a price, but the collector was not ready to commit to purchasing it immediately. Confident that the collector would purchase the work, the owner promised in a signed writing to sell the work to the collector at the agreed-upon price at any time before the end of the month. On the last day of the month, the collector sent the owner a check for the agreed-upon price, which the owner received on the following day. If the owner returns the collector's check and refuses to sell the artwork to the collector, which of the following best supports the owner's position that a contract had not been formed? A. The collector could not accept the owner's offer by mailing a check. B. The collector's acceptance of the owner's offer was not timely. C. The firm offer rule is not applicable because the collector was not a merchant with respect to the artwork. D. The firm-offer rule is not applicable because the owner was not a merchant with respect to the artwork.

B. The collector's acceptance of the owner's offer was not timely. *Owner made a firm offer to keep it open "until the end of the month." *Collector did not send acceptance until "last day of the month." Furthermore, the acceptance was not received until the day after. *Acceptance of a firm offer, option, or other irrevocable offer is effective only when it is received by the offeror. The MAILBOX RULE does NOT APPLY.*

A construction company contracted with a manufacturer to purchase 100 identical prefabricated windows to use while constructing houses in a gated community. The windows were to be delivered in shipments of 25 windows each on April 1, May 15, July 1, and August 15. The written contract, signed by both parties, was silent as to when payment for each shipment would be due. The manufacturer made the first two shipments in conformity with the contract requirements, and the construction company paid one-fourth of the full contract price upon each delivery. However, on June 1, the manufacturer demanded that the construction company pay the entire remainder of the contract price before the manufacturer made any further shipments. Which of the following statements is true? A. The construction company has no duty under the contract to make any payments until the final delivery is made. B. The construction company must pay the manufacturer one-fourth of the contract price upon delivery of each conforming shipment of windows. C. The construction company's failure to pay the requested sum will amount to a repudiation of the contract. D. The manufacturer waived his right to demand immediate payment of the full contract price when he accepted the first payment of one-fourt of the contract price on April 1.

B. The construction company must pay the manufacturer one-fourth of the contract price upon delivery of each conforming shipment of windows. *Special rules apply to installment contracts for the sale of goods (e.g., windows), which are governed by the UCC. Under the UCC, an installment contract is defined as a contract in which the goods are to be delivered in multiple shipments, and each shipment is to be separately accepted by the buyer.* **Payment by the buyer is due upon each delivery unless the price cannot be apportioned.** ***Under the UCC, an installment contract is defined as a contract in which the goods are to be delivered in multiple shipments, and each shipment is to be separately accepted by the buyer. Payment by the buyer is due upon each delivery unless the price cannot be apportioned.***

A jeweler and a goldsmith signed a written agreement that provided as follows: "For $3,000, the goldsmith shall sell to the jeweler a size six gold ring setting that the jeweler shall select from only the goldsmith's white gold ring designs." The agreement did not address any other specific terms with regard to the business arrangement between the jeweler and the goldsmith. When the jeweler arrived to select a ring, he refused to select one of the goldsmith's white gold ring designs. The jeweler claimed that the goldsmith, immediately prior to the execution of the written agreement, had orally agreed to broaden the jeweler's choices to also include rose gold ring designs. The jeweler also claimed that the goldsmith had, at the same time, orally agreed to include a set of earring settings, valued at $1,000, as an incentive for the jeweler's continued business. The goldsmith refused to sell to the jeweler any of his rose gold ring designs or include the earring settings. If the jeweler sues the goldsmith for damages, how should the court handle the evidence of the alleged oral agreements? A. The court should admit the evidence as to both the promise to include the earring settings and the option to choose a rose gold ring design. B. The court should admit the evidence as to the promise to include the earring settings but not the option to choose a rose gold ring design. C. The court should admit the evidence as to the option to choose a rose gold ring design but not the promise to include the earring settings. D. The court should exclude the evidence as to both the option to choose a rose gold ring design and the promise to include the earring settings.

B. The court should admit the evidence as to the promise to include the earring settings but not the option to choose a rose gold ring design. *The UCC presumes that a written contract is partially integrated. As a result, evidence that supplements the written contract is admissible -- but evidence that contradicts the writing is inadmissible -- under the parol evidence rule.*

A dancer signed a contract with a traveling circus to travel and perform as an aerialist for six months. The contract provided that the dancer would be paid $500 per week and would be guaranteed employment for the full six months, with an option to renew the contract for the next traveling season. Excited for the opportunity to perform for a traveling circus, the dancer turned down an invitation to dance with a theatre group for the same time period as the circus contract. After two weeks of traveling and dancing for the circus, the dancer sprained her ankle and was briefly hospitalized for one week. The circus was forced to hire another aerialist. After an additional week, the dancer's doctor gave her approval to return to work, but the circus refused to honor the remainder of the contract. The dancer brought an action against the circus for breach of contract. If the dancer wants to recover the highest possible amount of damages, which of the following is the dancer's best legal theory? A. The dancer detrimentally relied on the contract by declining the other dancing job. B. The dancer's failure to perform for two weeks was not a material breach of the contract. C. The dancer's performance of the terms of the contract was impracticable given her injury. D. The dancing contract with the circus is legally severable into weekly units.

B. The dancer's failure to perform for two weeks was not a material breach of the contract. *Damages for substantial performance v. material breach 1) A party who substantially performs can recover on the contract even though the party has not rendered full performance. Substantial performance = Contract price - Cost to receive full performance 2) A party who commits a material breach by failing to substantially perform cannot recover under the contract. Breaching party can only recover in restitution for any benefit conferred on the nonbreaching party minus damages for the breach. material breach = benefit conferred - damages for breach.*

An honest dispute developed between a condominium owner and a plumber over whether plumbing installed in the kitchen and bathrooms of the condominium satisfied contractual specifications. If the plumbing met those specifications, the condominium owner would owe the plumber $15,000 under the terms of the contract. The condominium owner offered to pay the plumber $10,000 in satisfaction of the owner's contractual obligations if the plumber replaced the plumbing in the kitchen with another grade of pipe. The plumber accepted the condominium owner's offer. After the plumber replaced the kitchen plumbing, the condominium owner refused to pay the plumber. In a breach-of-contract action brought by the plumber, the fact finder determined that the plumbing originally installed by the plumber did satisfy the contract specifications. The fact finder also determined that the plumber and the condominium owner entered into a substitute agreement under which the owner failed to deliver the required performance. What is the maximum amount that the plumber can recover in damages from the condominium owner? A. $25,000 B. $15,000 C. $10,000 D. Nothing

C. $10,000 *Accord Agreement versus Substitute Contract: 1) Accord Agreement: a. Formation - party accepts different performance in satisfaction of original contract. b. Breach - sue under ORIGINAL CONTRACT or ACCORD AGREEMENT. 2) Substitute Contract: a. Formation - original contract discharged & new contract created. b. Breach - sue under SUBSTITUTE CONTRACT ONLY. *This was a SUBSTITUTE CONTRACT. Fact-Finder determined it was a Substitute agreement. Read prompt CAREFULLY.*

A butcher and a seller entered into a written contract for the purchase and sale of a building to be used as a butchery. The closing was scheduled for June 1. On May 25, the seller was notified by the city that the building, which had previously been used as a butchery, had a number of significant city code violations. The seller immediately contracted with an electrician and others to correct the issues. Despite his best efforts, the seller realized that the building would not be brought up to code until at least June 10. The seller promptly sent written notification of this issue to the butcher and informed him that he would be unable to take possession of the building until June 10. Based on his agreement with the seller, the butcher had declined to renew his lease at his current location and was forced to remove his equipment and inventory from that location by the end of May. This included a large freezer that the butcher regularly used to store his inventory. Between June 1 and June 10, to prevent spoilage of his inventory due to the delay, the butcher had to rent space to store his equipment and inventory. He moved his freezer to the rented space to store his meat at a cost of $200 per day, plus the cost of electricity to run the freezer. The cost of electricity to operate the freezer would have been the same no matter where the freezer had been stored. On June 10, the building was up to code. The butcher paid the seller the agreed-upon purchase price of $300,000 and took possession of the property. What damages, if any, may the butcher recover from the seller? A. Nothing, because the seller acted in good faith. B. Nothing, because the contract did not contain a "time is of the essence" clause. C. $2,000 for the rental cost the butcher had to pay from June 1 to June 10. D. $2,000 plus the cost of electricity to run the freezer.

C. $2,000 for the rental cost the butcher had to pay from June 1 to June 10. *Expectation damages are computed as loss in value + other loss − costs avoided − loss avoided. For late delivery in real estate contracts, loss in value is measured by the fair market rental value of the property for the time that the buyer was denied possession.*

A couple, who wanted to open a pet grooming and supply store, contracted with a developer to lease space in a small strip mall that the developer was constructing. The lease was to begin on July 1, but on June 20, the developer informed the couple that the mall would not be finished, nor would the space be available, until August 1. The developer indicated that the first month's rent would be waived but that, because the lease did not contain a liquidated damages clause, he was not responsible for any damages attributable to the delay. As a consequence of the delay, the couple incurred storage costs and additional advertising expenses of $3,000. They also estimated in good faith that they lost $10,000 in sales. Which of the following amounts is the couple entitled to recover from the developer for the delay? A. Nothing, because the damages suffered by the couple's new business are too speculative. B. Nothing, because the lease did not contain a liquidated damages clause. C. $3,000, the amount incurred as a consequence of the delay. D. $10,000, the good-faith estimate of lost sales.

C. $3,000, the amount incurred as a consequence of the delay. *Incidental damages may be awarded as compensation for commercially reasonable expenses incurred because of the other party's breach. And consequential damages may be awarded to compensate for losses that do not flow directly and immediately from the other party's breach—so long as the losses are not too speculative.*

A maker of perfume contacted a manufacturer about supplying 1,000 readily available glass bottles for retail sales of perfume. The manufacturer offered to supply the bottles and to ship them within one week. The perfumer responded, "Ship them as soon as possible." The manufacturer shipped 1,000 bottles to the perfumer five days later. The perfumer accepted the bottles and filled them with perfume. Without waiting for the manufacturer's invoice, the perfumer sent a payment to the manufacturer based on a price of $2.50 per bottle. Prior to receiving this payment, the manufacturer sent the perfumer an invoice that reflected a charge of $3.50 per bottle. When the perfumer refused to pay $3.50 per bottle, the manufacturer returned the payment to the perfumer and initiated an action for the price. The court determined that a reasonable price for the bottles at the time of delivery was $3.25 per bottle. What amount should the court award the manufacturer per bottle? A. Nothing, because no contract was formed in the absence of a price term. B. $2.50, because the performer, as offeror, was master of the offer. C. $3.25, because this was a reasonable price for the bottles at the time of delivery. D. $3.50, because the manufacturer, as offeror, was master of the offer.

C. $3.25, because this was a reasonable price for the bottles at the time of delivery. *UCC generally requires Quantity to be specified in a contract - but otherwise the UCC fills the gaps with what is "reasonable" If (1) no price, (2) no time of delivery, (3) no payment, or (4) no place for delivery specified: 1) reasonable price; 2) reasonable time; 3) due at time and place buyer receives goods; 4) seller's place of business. Therefore, because price term was omitted, the UCC supplies a reasonable price at the time of delivery.

A grocery chain whose main customer base was families with young children contacted a cereal manufacturer. After various discussions regarding the cereal and the box, the two parties entered into a written contract whereby the grocery chain agreed to purchase 10,000 boxes of children's cereal on a monthly basis for $5,000, due upon delivery. The contract further stated that the cereal would be shaped like donuts, and each piece of cereal would be one of the seven colors of the rainbow. Finally, in listing the primary ingredients, the contract stated that the cereal would contain high-fructose corn syrup as a sweetener. When the first shipment arrived, the grocery chain refused to pay the $5,000 and repudiated the contract. The cereal manufacturer sued the grocery chain for damages and introduced the contract between the two parties into evidence. The grocery chain then attempted to offer evidence regarding the discussions that occurred between the two parties prior to the execution of the contract. It claimed that the cereal manufacturer had orally agreed to use evaporated cane juice, not high-fructose corn syrup, as the sole sweetener and that the cereal manufacturer would also include a small prize in each cereal box at a cost of a penny a box. In deciding whether to admit evidence of the oral agreement, how will the court likely rule? A. Neither the evidence regarding the use of evaporated cane juice as a sweetener nor the evidence regarding the small prize in each cereal box is admissible. B. Only the evidence regarding the use of evaporated cane juice as a sweetener is admissible. C. Only the evidence regarding the small prize in each cereal box is admissible. D. Both the evidence regarding the use of evaporated cane juice as a sweetener and the evidence regarding the small prize in each cereal box are admissible.

C. Only the evidence regarding the small prize in each cereal box is admissible. *The UCC presumes that a writing is partially integrated, so extrinsic evidence of additional consistent terms is admissible unless the parties "certainly" would have included the terms in the written contract. However, contradictory terms are not admissible.*

A homeowner entered into a written contract with a contractor to construct an elaborate tree house among the large trees located in the homeowner's backyard. After commencing construction of the tree house, the contractor discovered that one of the trees intended to be used as support for the tree house had a relatively common fungal infection in its core that would cause the strength of the tree's branches to falter if left untreated. Neither the homeowner nor the contractor had knowledge of the fungal infection when they entered into the contract, but the contractor knew that such infections were common in the area and did not request an inspection of the trees before entering the contract. The contractor also knew that treatment was available at a high cost, but that even after treatment, he would need to create additional heavy-load-bearing supports for the tree at a substantial cost. When the contractor informed the homeowner that he would not perform under the contract unless the homeowner provided at least 75% of the additional costs needed to make the structure safe, the homeowner refused to pay the additional amount. The homeowner then sued the contractor for breach of contract. What is the likely result? A. The contractor wins, because his performance was discharged due to impracticability. B. The contractor wins, because neither party was aware of the fungal infection. C. Th homeowner wins, because the contractor assumed the risk of the fungal infection. D. The homeowner wins, because the fungal infection did not render performance impossible.

C. Th homeowner wins, because the contractor assumed the risk of the fungal infection. *Impracticability Defense: 1. Unanticipated or extraordinary event made performance impracticable; 2. Contract was formed under basic assumption event would not occur; 3. Nonperforming party was not at fault in causing event to occur. *Nonperforming party must not have assumed risk that event would occur. -> contractor knew of fungal infection and how common it was.*

A homeowner entered into oral contracts with both a painter and a landscaper to perform services at his home. The landscaper was the first to begin the services, and shortly after he began to work, he realized that the projected cost of the project would increase dramatically. After the homeowner learned how high the cost of the landscaping services was going to be, he called the painter to tell her that he could not go through with their contract at that time. The painter stated that she had already purchased a standard set of paintbrushes to paint his home, as well as glass necessary to create a custom mosaic on a back corner of the house, according to the homeowner's specifications. She had also paid for a temporary city permit to park her utility van on the residential street where the homeowner lived. In a suit by the painter against the homeowner, which of the following is the painter LEAST likely to recover? A. The contract price minus the market cost of performance. B. The cost of the glass for the mosaic. C. The cost of the paintbrushes. D. The cost of the parking permit.

C. The cost of the paintbrushes. *emphasis on LEAST likely to recover; notice how the other choices are specifically tied to the job in question. A painter is generally expected to pay for paintbrushes for any job.*

A homeowner met with a number of general contractors regarding significant renovations on his home. After a few successful meetings with one particular contractor, the homeowner received a letter from the contractor stating that he would perform the renovation work for $10,000. The homeowner responded with a letter stating that he would "only pay $8,500 and no more." Upon receiving the homeowner's letter, the contractor immediately bought the materials necessary to complete the renovation and began renovating the homeowner's home. One week after beginning work on the homeowner's home, the contractor realized that the renovation would cost $1,600 more than he had anticipated due to the homeowner's very particular aesthetic requirements. After being informed of the higher cost, the homeowner refused to pay the contractor any amount over $8,500. The contractor promptly discontinued working on the renovation. The homeowner subsequently brought a breach-of-contract action against the contractor, seeking damages. Which party will likely prevail? A. The contractor, because the homeowner's offer could only be accepted by a return promise. B. The contractor, because the unexpected costs discharged his obligation to complete performance. C. The homeowner, because his offer was effectively accepted by commencement of performance. D. The homeowner, because the unilateral contract between the parties is enforceable.

C. The homeowner, because his offer was effectively accepted by commencement of performance. *An offer is presumed to be bilateral and can be accepted with a return promise or by starting performance. In contrast, a unilateral offer can only be accepted by completing performance.* **here, it was a bilateral contract that the contractor accepted by *starting* performance**

While at lunch with two coworkers, the owner of a bike offered to sell it to his manager for $100. The manager replied, "That ancient thing? I'll give you $50 for it." The owner's assistant then stated, "I'll buy it for $75." The owner immediately shook the assistant's hand and said "okay." At the same time, the manager said, "Wait a minute. How old is the bike?" When the owner replied, "Two years old," the manager said, "Okay, I'll buy it for $100." Which of the following statements is true? A. The owner may accept the manager's offer of $50. B. The owner may sell the bike to the manager for $100. C. The owner must sell the bike to the assistant for $75. D. The owner must sell the bike to the manager for $100.

C. The owner must sell the bike to the assistant for $75. *A counteroffer is a return offer made by the offeree to the offeror that relates to the same matter as the original offer but materially changes its terms. A counteroffer operates as both a rejection of the original offer and a new offer.* **Here, the owner offered to sell the bike to the manager for $100. The manager's reply ("I'll give you $50 for it") was a counteroffer that rejected and terminated the owner's original offer to sell the bike for $100, so the manager could no longer accept it (Choice D). However, the assistant's subsequent statement ("I'll buy it for $75") was a new offer, which the owner immediately accepted. This formed a valid contract, so the owner must sell the bike to the assistant for $75.**

A maker of handwoven rugs contracted with a supplier to provide yarn made from sheep's wool. The written contract specified that, for four years, the supplier would provide the rugmaker with 2,000 spools of yarn made from 100% sheep's wool per month, at $10 per spool, for a total of $20,000. Two years into the contract, the supplier sent the rugmaker 2,000 spools of yarn made from 90% sheep's wool and 10% synthetic fiber. The rugmaker sent the supplier a check for $15,000 for the shipment, and added a clear note on the check stating that the payment was in full for the shipment but was $5,000 less due to the synthetic fiber in the yarn. The supplier promptly deposited the check, and then four months later filed suit against the rugmaker for the remaining $5,000. The supplier has submitted evidence of the written contract, and the rugmaker has submitted evidence of the deposited check. What is the rugmaker's best defense in this situation? A. By depositing the check, the supplier was estopped from claiming that the rugmaker owed him an additional $5,000. B. The rugmaker's and supplier's good faith dispute over the yarn composition suspended the rugmaker's obligation to pay the remaining $5,000. C. The supplier deposited the check for $5,000 less than the contract price, thereby discharging the rugmaker of any further duty to pay the remaining amount for that month's shipment. D. The supplier's act of knowingly depositing the check for $15,000 was a novation that relieved the rugmaker from any further liability.

C. The supplier deposited the check for $5,000 less than the contract price, thereby discharging the rugmaker of any further duty to pay the remaining amount for that month's shipment. *Accord/Satisfaction Rule: the contractual obligation will be discharged by accord and satisfaction, if the party tendered a negotiable instrument with a conspicuous statement that it was tendered payment in full, and the other party obtained payment of the instrument.*

A library contacted a local artist expressing an interest in purchasing a particular one of the artist's sculptures for display at the library. The library's agent and the artist executed a written contract that was signed by both parties and provided that the library would purchase the sculpture for $1,000 due upon delivery of the sculpture to the library. Just before they signed the contract, the agent told the artist, "Plan on delivering the sculpture in 10 days, but please remember that the library's obligation to purchase the sculpture will be conditioned on the approval of the chairperson of the Artistic Patronage Council, as it will be providing the library with the funds for this sale." The chairperson of the Artistic Patronage Council orally approved the sale the next day. However, 10 days after the contract was executed, the artist decided that he did not want to sell the sculpture. If the library sues the artist for breach of contract, is the library likely to prevail? A. No, because the library's agent made an illusory promise. B. No, because there was no mutuality of remedy when the contract was executed. C. Yes because the agreement was supported by good consideration even though it was conditioned on an uncertain event. D. Yes, because the artist waived any lack of consideration by signing the contract.

C. Yes because the agreement was supported by good consideration even though it was conditioned on an uncertain event. *A contract must generally be supported by valuable consideration, as evidenced by a bargained-for change in the legal position between the parties (i.e., "mutuality of consideration"). Consideration can take the form of a return promise to do (or not do) something or the actual performance (or nonperformance) of some act. Such performance can be made contingent upon a condition.* **Condition precedent & Condition subsequent Condition precedent: where a party's duty to perform arises upon the occurrence or nonoccurrence of an uncertain future event (i.e., the event creates the duty) Condition Subsequent: Where a party's duty to perform is released upon the occurrence or nonoccurrence of an uncertain future event (i.e., the event extinguishes the duty)** ***watch out for illusory promises. (promises that essential promise nothing -- breach isn't possible)***

On April 1, a buyer and a seller executed a written contract for the sale of an antique car for $40,000, delivery on May 1. The contract contains a clause indicating that it is a total integration of the parties' agreement. As they each signed the contract, the buyer orally reminded the seller that the buyer's duty to purchase the car was conditioned on his ability to get approval for a loan by April 20 to fund the purchase. The seller orally agreed, though the condition was not noted in the written contract. When the seller contacted the buyer to execute the sale on May 1, he discovered that the buyer had attempted but failed to get a loan and could not afford to purchase the car. The buyer refused to honor the contract. If the seller sues the buyer for breach of contract, will the court likely admit the evidence of the oral condition regarding the buyer's approval for a loan? A. No, because the oral agreement contradicts the terms of the written contract. B. No, because the written contract is a complete integration of the agreement between the parties. C. Yes, as proof of a condition precedent to the buyer's obligation under the contract. D. Yes, because the oral agreement was a distinct and separate contract.

C. Yes, as proof of a condition precedent to the buyer's obligation under the contract. *Evidence used to establish a condition precedent that must occur before a contract becomes effective is admissible under an exception to the parol evidence rule.*

On December 1, a candy store sent an order to a jelly bean manufacturer for the shipment of 500 bags filled with red, heart-shaped jelly beans in anticipation of Valentine's Day on February 14. The order stated that the jelly beans should arrive no earlier than January 15 in order to ensure their freshness, but no later than February 1 in order to maximize sales before Valentine's Day. The order also clearly stated in bold lettering that the candy store could cancel the order at any time before December 15. The jelly bean manufacturer mailed an acceptance of the order, which was received by the candy store on December 3. The candy store did not cancel the order before December 15. On January 20, the candy store received the 500 bags filled with red, heart-shaped jelly beans from the manufacturer in compliance with the order. However, the candy store refused to accept the jelly beans. If the manufacturer sues the candy store for breach of contract, will it be likely to prevail? A. No, because the candy store's offer was an illusory promise. B. No, because the manufacturer's shipment was only an offer. C. Yes, because an enforceable contract existed. D. Yes, because the candy store's right to cancel the order was a condition subsequent.

C. Yes, because an enforceable contract existed. *An offer that is illusory will become legally binding if (1) circumstances change such that the offer imposes obligations on both parties and (2) the offer is accepted after the change in circumstances.*

A wedding planner contracted with a local bakery to make cupcakes for an upcoming wedding reception. The bakery was very experienced in making cupcakes and had a great reputation in the community. Although there were other comparably skilled cupcake makers in the area, the wedding planner eventually chose the bakery due to the price it quoted her for the cupcakes. A few months before the wedding reception, the bakery's head baker unexpectedly had to take a leave of absence to deal with a medical issue. The bakery subsequently assigned the contract to a pastry chef in the same community. The pastry chef also had an excellent reputation in the community and was at least equally as skilled at making cupcakes as the bakery. The bakery told the wedding planner about this assignment, and the wedding planner did not object. When the pastry chef fully performed on the contract and delivered the cupcakes, which conformed to the contract requirements, the wedding planner refused to accept or pay for the cupcakes. On these facts, has the wedding planner breached the contract? A. No, because the bakery breached the contract first by assigning the contract to another party. B. No, because the wedding planner only had a duty to accept performance by the bakery. C. Yes, because the assignment of the contract was permitted and the pastry chef property and fully performed. D. Yes, because the bakery told the wedding planner about the assignment of the contract and the wedding planner did not object.

C. Yes, because the assignment of the contract was permitted and the pastry chef property and fully performed. *Delegation of contractual duties is not permitted when (1) the contract prohibits delegation; OR (2) the other party to the contract has a substantial interest in having the delegating party perform. Otherwise, delegation is permitted, and the other party to the contract must accept performance by the delegatee.*

After the death of a farmer, the executor of his estate held an auction sale of his farm equipment. The executor specified that she reserved the right to withdraw any item from the sale. A neighbor placed a bid on a tractor. The bid was acknowledged by the auctioneer. Before another bid was placed, and before the auctioneer announced the completion of the sale, the neighbor informed the auctioneer that he was withdrawing his bid. Must the auctioneer permit the neighbor to withdraw his bid? A. No, because the auctioneer may, but is not required to, accept the neighbor's withdrawal of his bid. B. No, because there is no right to withdraw a bid. C. Yes, because the auctioneer had not announced the completion of the sale. D. Yes, because the seller retained the right to withdraw any item from the sale.

C. Yes, because the auctioneer had not announced the completion of the sale. *At a reserve or no-reserve auction, a bidder has the right to withdraw a bid until the auctioneer announces the completion of the auction sale.*

A boutique hotel contracted with a seamstress to handmake 500 pillows. The signed contract specified that the pillows should be filled with down and that the pillow covers should be made with white, 1000-thread-count cotton fabric. Before the seamstress began making the pillows for the boutique hotel, she secured another commission for work that would prevent her from making the hotel's pillows. As a result, the seamstress informed the boutique hotel that she was passing on the hotel's contract to her former business partner, who was comparable in talent and skill at making high-quality pillows. The boutique hotel did not object to the substitution. The former partner diligently worked on making the pillows, using white, 1000-thread-count fabric to make the pillow covers. However, instead of using down to fill the pillows, she used a comparably priced synthetic microfiber. The boutique hotel subsequently filed a breach-of-contract action against the seamstress. Will it succeed? A. No, because the former partner's use of a synthetic microfiber instead of down did not reduce the value of the pillows. B. No, because the boutique hotel was aware of and did not object to the delegation of the seamstress's duties to her former partner. C. Yes, because the boutique hotel had not released the seamstress from liability under the contract. D. Yes, because the seamstress did not give consideration for delegating the contract to the former partner.

C. Yes, because the boutique hotel had not released the seamstress from liability under the contract. *Generally the obligations under a contract can be delegated to another, however, this does not release the delegator from liability UNLESS there is a novation. Here, the hotel did not object to the delegation nor did they expressly imply a release of the seamstress from liability.*

In June, a local chef learned of a new business that opened in the area. Hoping to attract the business as a new client, the chef sent the business an offer consisting of a catalog of menus available through his catering service and a form letter that he sent to all new businesses in the area. The letter was signed by the chef and included the following language: "Welcome! I specialize in creating delicious meals with local and organic ingredients, and I would be honored to be your catering source for all your business, promotional, and personal needs! To welcome you to the community, I would like to offer you a 25% discount off my catalog prices on any three-course meal order, for up to 100 people, submitted this calendar year. I hope to hear from you soon, and I look forward to doing business with you!" No communication occurred between the parties until the end of November, when the business faxed an order form to the chef requesting a catered meal for 60 people at a promotional event for a 25% discount. The chef refused to provide the business with the catered meals at a 25% discount. As a result, the business sued the chef for breach of contract. The court found that both parties are merchants with respect to this transaction. Is the business likely to succeed in its action? A. No, because the business's power of acceptance terminated after a reasonable period of time. B. No, because the form letter was only an invitation to deal. C. Yes, because the chef had not revoked the offer before the end of the calendar year. D. Yes, because the signed promotional letter created a firm offer.

C. Yes, because the chef had not revoked the offer before the end of the calendar year. *An offer is an objective manifestation by an offeror of a willingness to enter into an agreement that creates the power of acceptance in an offeree. The power to accept may be exercised until the offer is terminated.* **Termination can occur in many ways, including lapse and revocation. **An offer will terminate by lapse if it is not accepted by the time stated in the offer or, if no time is stated, within a reasonable time.** ***And a revocable offer will terminate by revocation if the offeror revokes the offer prior to acceptance.*** ****Here, the chef offered the business a 25% discount on any three-course meal order submitted within the calendar year.****

On March 1, a company contracted with a singer to perform for the company picnic on May 1 for a fee of $10,000. On March 17, the singer informed the company that she had signed a contract to film a movie. She suggested that the company hire another singer to take her place at the picnic. On April 1, the company hired the recommended replacement singer to perform at its picnic for $15,000. On April 25, the original singer informed the company that she had decided not to take the movie deal and will be available to perform on May 1. The original singer arrived at the picnic on May 1 ready to sing, but the company let the replacement singer perform. The company refused to pay $10,000 to the original singer. Is the company likely to prevail in a breach-of-contract claim against the original singer? A. No, because the company prevented the original singer from fulfilling her contractual obligation by refusing to let her perform on May 1. B. No, because the original singer retracted her repudiation before the scheduled performance. C. Yes, because the company hired the replacement singer as a substitute for the original singer before she retracted her repudiation. D. Yes, because the replacement singer's consent to the delegation of the original singer's duties did not create a novation.

C. Yes, because the company hired the replacement singer as a substitute for the original singer before she retracted her repudiation. *A repudiation of contractual duties may be retracted before the nonrepudiating party: (1) cancels the contract; (2) materially changes position in reliance on the repudiation; OR (3) indicates that he/she considers the repudiation to be final.*

An employee signed an employment contract with a company to be a remote salesman. The contract set forth the employee's yearly salary and a requirement that he work at least 30 hours per week. The contract also required the employee to log his monthly sales figures online by the last business day of the month. However, the contract did not provide monthly sales goals and made no guarantees about how or when a potential bonus would be paid or earned. After the employee worked remotely for the company for a few months, the company called the employee on March 15. Over the phone, a representative of the company explained that the company was establishing a new bonus program for employees who reached certain monthly sales goals. The representative explained that the employee would receive a bonus for reaching these sales goals, provided that he not only logged his sales online but also called the company's central office on the first Friday of each month during business hours and reported his sales figures for the previous month. The employee worked diligently for the remainder of March to meet the sales figures required to receive a bonus, working approximately 40 hours per week. He logged his sales online at the end of March, as required by his contract. On the first Friday in April, the employee called the company to report his March sales figures. However, the company had disconnected its phone lines and did not receive the employee's call. The company subsequently refused to pay the employee a bonus for meeting his sales goals in March. Will the employee likely prevail in a breach-of-contract action against the company for his unpaid March bonus? A. No, because the employee failed to notify the company during his conversation on March 15 that he had accepted the offered bonus plan. B. No, because the employee's performance of his preexisting legal duty to sell merchandise for the company will not qualify as consideration. C. Yes, because the company itself prevented the employee from satisfying the express condition of the March 15 offer. D. Yes, because the employee worked for 40 hours per week for the month of March.

C. Yes, because the company itself prevented the employee from satisfying the express condition of the March 15 offer. *When nonoccurrence of condition is excused: 1) Waiver 2) Wrongful interference 3) Estoppel 4) Disproportionate forfeiture* **Since the company wrongfully prevented the employee from reporting his sales figures, the doctrine of prevention will preclude the company from raising the nonoccurrence of that condition as a ground to avoid paying the bonus. Therefore, the employee will likely prevail in this breach-of-contract action.** ***A condition's nonoccurrence is excused when the party whose duty to perform is subject to a condition wrongfully prevents or interferes with the occurrence of that condition.***

A widow offered to sell her small business, together with all of the business's assets, to a nonprofit organization. The organization accepted, and on June 1, it signed and executed a contract providing for the sale of the business for $25,000 at the end of the month. When the organization's agent signed the contract, she orally informed the widow that the organization's duty to purchase the business was conditioned on obtaining approval from a local zoning board to convert the business's primary office into an affordable-healthcare clinic. A week later, the woman received another offer to purchase her business for $35,000. At the end of the month, seeking to accept the other offer, the widow refused to honor the contract with the organization because it had neglected to request the necessary approval from the zoning board. The organization sued the widow for breach of contract. The organization presented clear evidence that it had the necessary funds to perform on the contract at the end of the month, and that the zoning board would have routinely approved the organization's plans for the office. Is the organization likely to prevail in its action against the widow? A. No, because the express condition of the zoning board's approval had not occurred by the end of the month. B. No, because the organization's failure to seek approval from the zoning board was a repudiation of the contract. C. Yes, because the condition of approval by the zoning board has been waived by the organization. D. Yes, because the condition of approval by the zoning board was not included in the written contract.

C. Yes, because the condition of approval by the zoning board has been waived by the organization. *A party to a contract whose duty to perform is subject to a condition can waive the condition by words or conduct.*

A painter entered into a contract with a homeowner to paint the exterior of the homeowner's home over a weekend while the homeowner was on vacation. After the homeowner left on his vacation, the painter was offered a second job that paid slightly more during the same weekend. The painter delegated his duty under the first contract to a second painter in exchange for a $50 advance and a promise to split his profits from both jobs with the second painter. The second painter took the advance and agreed to paint the homeowner's home that weekend. The first painter worked on the second job that weekend and did not check on the homeowner's home. When the homeowner returned from vacation, he discovered that his home had not been painted at all. In addition, no one has been able to locate the second painter. Does the homeowner have a valid cause of action against the first painter? A. No, because the homeowner has not exhausted his remedies against the second painter. B. No, because the second painter accepted consideration in exchange for his promise to paint the home. C. Yes, because the first painter was not released from his liability to the homeowner. D. Yes, because the homeowner is unable to recover from the missing second painter.

C. Yes, because the first painter was not released from his liability to the homeowner. *When contractual obligations or duties are delegated, the delegator remains liable under the contract unless the other party to the contract expressly or impliedly agrees to release that party and substitute a new one (i.e., novation).*

A sister convinced her brother that they should open a small coffee shop. Their friend, a guitarist, suggested bringing his band to play live music in order to attract customers. He did not request any payment, saying that the publicity would be good for the band. The siblings agreed, and the band started playing at the coffee shop weekly. The coffee shop became a success, in no small part due to the band's performances. When a businessperson offered to buy the coffee shop from the siblings, they orally agreed to each pay the guitarist $10,000 out of their share of the sale proceeds for his help in making the shop popular. The sister told the guitarist about their agreement. He was so delighted with it that he made a down payment on a new car. By the time the sale of the coffee shop was finalized, the brother had encountered financial difficulties. After the sale, the siblings signed a written contract stating that the sister would pay the guitarist $10,000 and her brother would pay him $5,000. If, after the sale, the brother pays the guitarist only $5,000, will the guitarist have a valid basis for an action against the brother for another $5,000? A. No, because the guitarist was bound by the written modification of the contract made by the siblings. B. No, because the guitarist was only a donee beneficiary of the oral contract between the siblings. C. Yes, because the guitarist's reliance on the promised payment prevented the siblings from changing the obligations of their oral contract. D. Yes, because the oral promise to pay $10,000 to the guitarist was made binding by the guitarist's valuable and uncompensated contributions to the business.

C. Yes, because the guitarist's reliance on the promised payment prevented the siblings from changing the obligations of their oral contract. *Intended beneficiary: Rights vest when the beneficiary: 1) detrimentally relies on the rights created; 2) manifests assent to the contract at one party's request; OR 3) files a lawsuit to enforce the contract. *guitarist's rights vested when he detrimentally relied on what the sister said and made a down payment on a new car.

On May 1, a clothing manufacturer sent a written offer to a retailer for the sale of 1,000 pairs of designer jeans at a price of $50 per pair, including delivery and transportation costs. The proposed delivery date was June 15. On May 5, the retailer mailed a letter accepting the manufacturer's offer. On May 15, a natural disaster occurred, causing fuel prices to significantly increase. As a result, the manufacturer sent a letter to the retailer requesting an additional $500 to cover the increased delivery and transportation costs for the shipment of the jeans. The retailer considered seeking out a new supplier. However, the retailer hoped to continue doing business with this manufacturer in the future, so she decided against it. The retailer returned a signed letter promising to pay the additional $500. On June 15, the manufacturer delivered 1,000 pairs of jeans to the retailer. The retailer paid the manufacturer $50,000 but refused to pay the additional $500. Can the manufacturer enforce the retailer's promise to pay an additional $500? A. No, because no consideration supported the promise to pay the additional $500. B. No, because the attempted modification was unconscionable. C. Yes, because the manufacturer made its request for the additional $500 in good faith. D. Yes, because the manufacturer relied on the promise when it delivered the jeans to the retailer.

C. Yes, because the manufacturer made its request for the additional $500 in good faith. *Under the UCC, no consideration is needed to modify a contract. All that is required is good faith—i.e., honesty in fact and fair dealing per reasonable commercial standards.*

An organic-produce supplier sent her produce catalog to a local restaurant on April 15. The catalog came with a signed letter stating: "I will supply you with as many of the items in the enclosed catalog as you order before August 1 of this year, and I assure you that this offer and the prices provided in the catalog will remain firm until August." The supplier received no reply. In June, the supplier's tomato crop was infested with a fungus that decimated her harvest. She was left with half the tomatoes she had expected to harvest. On June 15, the supplier sent the restaurant a signed letter stating that the price for the tomato crop was now twice the price listed in the catalog. On July 1, the restaurant sent the supplier an order for tomatoes but demanded the tomatoes at the price listed in the catalog. Has a contract been formed for the sale of the tomatoes to the restaurant at the catalog price? A. No, because the destruction of the tomatoes supports a defense of impracticability. B. No, because the restaurant offered no consideration to make the original offer irrevocable. C. Yes, because the original offer was irrevocable on June 15 and on July 1. D. Yes, because the restaurant exercised its power of acceptance within a reasonable time.

C. Yes, because the original offer was irrevocable on June 15 and on July 1. *Under the UCC, a firm offer is irrevocable and cannot be modified for a period not to exceed three months—even if a longer time period is stated or implied—unless the offeree gives consideration to validate it beyond the three-month period.*

A law student attended law school on a full scholarship. At the end of the law student's second year, she lost her scholarship. In order to fund her third year, she borrowed $50,000 from her rich uncle. They executed a written agreement stating that the law student would repay the loan two years from May 15, the date of her law school graduation. On May 15, two years after her graduation, the law student did not pay her uncle back because she had been unable to find a job as a lawyer. Instead, she was working as a server at a coffee shop. The uncle took no legal action. Four years later, the law student was still unable to pay the uncle back, but she did write him a signed letter stating that "I know I still owe you $50,000. I will repay you $50,000 if I get a law firm job." The statute of limitations for suits to collect debts in the jurisdiction is three years. Is the law student's promise contained in the letter to repay the loan enforceable? A. No, because the promise to repay is not supported by consideration. B. No, because the statute of limitations for collecting debts in the jurisdiction is three years. C. Yes, because the promise was made after the statute of limitations had run. D. Yes, because the uncle's foregoing of legal action constitutes consideration.

C. Yes, because the promise was made after the statute of limitations had run. *A new promise to pay a debt after the statute of limitations has run is enforceable without any new consideration.*

The owner of a high-rise building entered into a written contract with a company to maintain and service the elevators in the building. The written contract contained the following provision: "This contract is the entire and final agreement of the parties regarding the maintenance and servicing of the elevators in Building. It supersedes any prior agreements, understandings, or negotiations." On the starting date of the contract, the company discovered that the building's elevators were significantly older than the owner had orally represented to the company during the negotiations prior to the signing of the contract. The company refused to maintain and service the elevators unless the owner agreed to a sizable increase in the monthly payments called for in the contract. The owner refused and found another entity to maintain and service the elevators at a cost below what the company wanted but above the original contract price. The owner then sued the company for breach of contract, seeking the difference between the contract price and the amount paid to the entity that was currently providing elevator maintenance and service. At trial, the company seeks to introduce evidence of the owner's oral statement as to the age of the elevators during contract negotiations. Should the court permit the introduction of this statement? A. No, because of the parol evidence rule. B. No, because the contract for services is governed by common law. C. Yes, because the statement relates to a contract defense. D. Yes, because the statement was oral, not written.

C. Yes, because the statement relates to a contract defense. *The parol evidence rule does not bar evidence of prior or contemporaneous communications between contracting parties when the evidence is offered to establish a defense to contract formation (e.g., misrepresentation).*

An owner of a nail salon started a new promotion to boost sales. The owner advertised that she would pay $100 to any customer who purchased a new type of long-lasting manicure from her nail salon over the next four months if the customer's manicure did not stay intact for an entire month. A woman purchased the advertised manicure during the promotional period per the terms set forth by the owner, but her manicure only stayed intact for three weeks. The woman tried to collect the $100 from the owner, but the owner refused to pay her. Can the woman collect the $100? A. No, because the woman did not notify the owner that she was accepting the offer prior to purchasing the manicure. B. No, because there was no bargained for exchange. C. Yes, because the woman accepted the offer by purchasing the long-lasting manicure during the promotional period. D. Yes, because this was an enforceable bilateral contract.

C. Yes, because the woman accepted the offer by purchasing the long-lasting manicure during the promotional period. *A unilateral contract arises when an offeror promises something in return for an offeree's complete performance of a specified act. Therefore, a unilateral contract is not formed until the offeree's performance is fully completed.*

On October 1, a retail sporting equipment store telephoned a shoe manufacturer and offered to buy a minimum of 50 and a maximum of 100 pairs of running shoes at $40 a pair to be delivered in 60 days. The manufacturer orally accepted the offer and then immediately faxed a signed letter to the store. The letter contained the following language: "This letter confirms our agreement by telephone on October 1 to sell you 50 pairs of running shoes for 60-day delivery." Sixty days later, the manufacturer delivered 100 pairs of conforming running shoes to the store. However, the store rejected all 100 pairs of shoes because it had found the same shoes from another seller for $30 a pair. Due to lack of demand, the manufacturer cannot resell the 100 shoes without suffering an economic loss. Can the manufacturer enforce a contract against the store? A. No, because the manufacturer did not state the agreed upon price term in the faxed letter. B. No, because the store did not sign the faxed letter. C. Yes, for the sale of 50 pairs of shoes, because the manufacturer's faxed letter stated that quantity term. D. Yes, for the sale of 100 pairs of shoes, because the store wrongfully rejected that quantity of property delivered and conforming shoes.

C. Yes, for the sale of 50 pairs of shoes, because the manufacturer's faxed letter stated that quantity term. *Battle-of-the-forms rule: generally an acceptance with additional terms (counter-offer) is a valid acceptance; but whether the terms are included depend on: 1. if both parties are merchants; 2. if the offer doesn't expressly limit the terms in the offer; 3. if the new terms do not materially alter the original contract; 4. and the offeror does not object within a reasonable amount of time (to the new terms).* *The store's signature is not necessary here because of the MERCHANT EXCEPTION - only one signature is needed and the store had notice to object within 10 days.* [statute of frauds]

A tenant rented a small cabin from a landlord. The lease provided that the tenant was permitted to make structural improvements to the cabin but that the tenant must pay for such improvements. Relying on this clause in the contract, the tenant contacted a contractor to install a loft in the cabin for $10,000. The tenant and the contractor agreed in a writing signed by both parties that payment would be due 30 days after the loft was completed. The contractor knew that the tenant was renting the cabin and sent the landlord a letter informing him of the impending construction on his property. The landlord received the letter and did not reply. The contractor completed the loft, which increased the market value of the cabin by $6,000. Ten days later and three months before the end of her lease, the tenant vacated the cabin and disappeared. Thirty days after the loft was completed, the contractor's bill remained unpaid. If the contractor has no remedy quasi in rem under the jurisdiction's mechanic's lien statute, which of the following will give the contractor the best chance of recovery in personam against the landlord? A. An action as a third-party beneficiary to the lease. B. An action based on an implied-in-fact contract. C. An action based on promissory estoppel. D. An action in quasi-contract for the benefit conferred on the landlord.

D. An action in quasi-contract for the benefit conferred on the landlord. *A plaintiff can recover under a quasi-contract theory - despite having no contractual relationship with the defendant - if the plaintiff conferred a non-gratuitous benefit on the defendant that resulted in unjust enrichment.*

A recent college graduate offered to buy all of the computers from a failing online retailer for which he had been an intern during college, and the retailer accepted. The terms of the written agreement were such that the graduate would pay $10,000 for a "reasonable number of computers" since the retailer was winding up its business and no longer needed them all. Due to his internship with the retailer, the graduate knew that there were 50 computers in the office and that nearly all of them were unused, so he believed that he would receive all 50 computers once the retailer closed. He gave the retailer a check for $10,000 and, in return, took 10 computers from the office that day. With the help of the $10,000 and a sudden upswing in the online retail market, the retailer became profitable. When the graduate demanded the remaining 40 computers, the retailer refused. Instead, the retailer returned the $10,000 to the graduate and demanded the return of the 10 computers that were in the graduate's possession. The graduate sued the retailer for breach of contract. The retailer has moved to dismiss the suit, arguing that no valid contract existed. How is the court likely to rule? A. Deny the motion, because the court may supply missing terms in a contract. B. Deny the motion, because the parties formed a requirements contract. C. Grant the motion, because the retailer's increased profitability constituted a supervening event. D. Grant the motion, because there was no agreement as to quantity.

D. Grant the motion, because there was no agreement as to quantity. *Under the UCC, a contract must specify a quantity that is certain or determinable by reference to objective facts, such as the buyer's actual requirements or the seller's actual output (requirements and output contract). If it does not, the contract fails for indefiniteness.* *A "reasonable number" is not an objective fact through which the quantity term can be determined. And since the court cannot supply the missing quantity term, the terms were too indefinite to form a contract.*

An independent trucker and a manufacturer entered a written contract for the delivery of a farming implement from the manufacturer to a farmer. Under the terms of the contract, the trucker promised "to deliver a farming implement from the manufacturer to the farmer," and in exchange, the manufacturer promised "to pay the trucker if the trucker delivers the implement directly to the farmer after picking it up." The trucker picked up the implement but, instead of driving directly to the farmer, drove 100 miles out of his way to pick up another item from a third party before delivering the implement to the farmer. The manufacturer, unaware that the trucker had failed to deliver the implement directly to the farmer, refused to pay the trucker. Who has breached this contract? A. Both the trucker and the manufacturer B. The trucker only. C. The manufacturer only. D. Neither the trucker nor the manufacturer.

D. Neither the trucker nor the manufacturer. *If contracting parties expressly agree to a condition precedent—an uncertain future event that must occur before a party's obligation to perform arises—then performance is not due until the condition is fully satisfied.*

A student inherited a large tract of undeveloped land from an eccentric uncle. The student had no present need for the land, and because he had numerous student loans, he decided to sell the land. He advertised a proposed sale of the property, and he was soon contacted by a rancher who owned property adjacent to the offered land. The rancher wanted to purchase the student's property to expand his ranch and to build facilities for dairy production. The student told the rancher that his car had just broken down and that he was eager to sell the property quickly so that he could repair his car for his commute to class. Although the rancher was fully aware of the fair market value of the property, he offered the student a cash price 80 percent less than the property was worth. The student, disappointed with the low price but desperate to repair his car, accepted the rancher's offer. On these facts, which of the following legal concepts would give the student the best chance of canceling the contract with the rancher? A. Bad faith. B. Duress. C. Equitable Estoppel D. Unconscionability

D. Unconscionability *A court may modify or refuse to enforce a contract on the ground that it is unconscionable. A contract is unconscionable when it is so unfair to one party that no reasonable person in that party's position would have agreed to it.*

A mining company contracted with a railroad to transport 10,000 tons of coal from the company's mines to a power company at a cost of $100,000. The railroad told the mining company that the coal would arrive at the power company on June 1, but the contract contained a clause that the railroad would not be liable for any losses suffered by the mining company as a result of a late shipment. The railroad was aware that the mining company had contracted with the power company to deliver the coal on June 1, and pursuant to standard industry custom, the price to be paid by the power company decreased by $1 per ton for each day that the coal was late. The shipment of coal did not reach the power company until June 11, and the railroad had no justification for the 10-day delay. Because of the delay, the mining company lost $100,000 in revenue from the sale. The mining company filed suit against the railroad for breach of contract, claiming $100,000 in damages. Is the mining company likely to succeed in its claim? A. Yes, because the damages that the mining company would suffer from the railroad's delay were known to the railroad prior to shipment of the coal. B. Yes, because consequential damages cannot be excluded by a merchant. C. No, because the claimed damages are disproportionate to the original contract price between the railroad and the minding company. D. No, because the contract between the mining company and the railroad protected the railroad from liability for losses suffered by the mining company due to a late shipment.

D. No, because the contract between the mining company and the railroad protected the railroad from liability for losses suffered by the mining company due to a late shipment. *parties to a contract are free to set their own terms, including limiting a party's liability for consequential damages*

A homeowner called and entered into an oral contract with an engineer to build a retaining wall at his home. The engineer had recently created her own professional website to advertise her services. Two days after their conversation, but before the engineer began work on the wall, the homeowner lost his job. As a result, the homeowner immediately called the engineer to tell her that he could not go through with their contract at that time. The engineer stated that she had already purchased materials for the job. She had also paid for a temporary city permit to park the necessary equipment on the street where the homeowner lived and hired a photographer to take pictures of the finished wall for her website. Which of the following would NOT be a possible liability for the homeowner? A. The contract price minus the market cost of performance. B. The cost of the materials. C. The cost of the permit. D. The cost of the photographer.

D. The cost of the photographer.

A company leased office space in a downtown building and subsequently entered into a written contract with a supplier to purchase furniture for the office. A dispute later arose over the tables and desks delivered by the supplier. The contract called for "cherry tables and desks" of designated designs. The company contended that the word "cherry" indicated the type of wood from which the tables and desks were made. The supplier, having delivered tables and desks made of a less expensive wood and finished with a cherry veneer, asserted that the use of the word "cherry" referred to the appearance of these items and did not require that the furniture be made solely of cherry wood. In the litigation of this dispute, the company sought to introduce a statement made by the supplier during negotiations that the tables and desks were of "solid-wood construction." In determining whether the parties intended the contract to be their final agreement, which of the following best reflects the rule of interpretation that the court should apply? A. The court can find that the contract is integrated only if it contains a merger clause. B. The court is permitted to look only within the "four corners" of the document for evidence of intent. C. The court must presume that the written contract is fully integrated. D. The court should presume that the written contract is partially integrated.

D. The court should presume that the written contract is partially integrated. *Under the UCC, a court should presume that a written contract for the sale of goods is only partially integrated. As a result, evidence of additional consistent terms is admissible unless the court concludes that the parties certainly would have included those terms in the writing.*

A man was moving to another state and decided that he wanted to give away some of his belongings. The man knew that his brother had always expressed interest in the man's antique desk. The man called the brother and said, "I'm going to be moving in two weeks. I would like to give you the antique desk as a gift. I'll drop it off at your house on my way out of town." The brother told the man that he was very grateful for the gift and was looking forward to having the desk in his home office. The brother, in reasonable reliance on the man's promise, immediately disposed of his old desk and made room for the antique one. A couple of days later, an appraiser, who was a friend of the man, visited the man's house for dinner. While at his house, the appraiser saw the antique desk and informed the man that it was worth well over $20,000. The man decided to keep the desk and did not drop it off at the brother's house on his way out of town. The brother brought suit against the man to recover the antique desk. If the court finds in favor of the man on these facts, what is the most likely reason? A. A promise to make a gift in the future cannot be enforced. B. The brother did not rely to his detriment on the man's promise. C. The man's promise was not in writing. D. The man's refusal to give the antique desk did not cause injustice.

D. The man's refusal to give the antique desk did not cause injustice. *Promissor Estoppel elements: 1) the promisor should reasonably expect the promisee to rely on the promise; 2) the promisee detrimentally relies on the promise; AND 3) injustice can be avoided only by enforcement of the promise.* **Therefore, if the court finds in favor of the man, the court must have concluded that the man's failure to give the antique desk to the brother as promised did not cause injustice.**

The owner of a bed and breakfast hired an artist to paint nature-themed murals in each of the five bedrooms. The contract provided that payment was due upon the satisfactory completion of all five rooms. The owner told the artist that each mural should relate to the name of the bedroom, but she otherwise gave the artist broad discretion in designing each mural. When the owner checked the artist's progress a few weeks later, she found that although the murals in the three completed rooms related to the theme of the rooms, the color choices clashed with the overall décor of the bed and breakfast. The owner told the artist that she would accept his performance on the first three rooms, but she asked him to incorporate a different color palette in the remaining rooms. The artist, unwilling to compromise his artistic autonomy, refused to paint the remaining two rooms and immediately terminated the contract. What is the artist entitled to recover from the owner of the bed and breakfast? A. Nothing, because the contract expressly provided that payment would be due upon the completion of all five rooms. B. Nothing, because the murals in the three completed rooms clash with the overall décor of the bed and breakfast. C. The artist's expenditures in painting the first three rooms and the artist's anticipated profit for painting the last two rooms. D. The reasonable value of the artist's services in painting the first three rooms, less any damages the owner may suffer from the artist's failure to paint the last two rooms.

D. The reasonable value of the artist's services in painting the first three rooms, less any damages the owner may suffer from the artist's failure to paint the last two rooms.

On January 5, a buyer and a seller contracted for the delivery of 100 widgets if they could be delivered by February 20. The agreement was made in a writing signed by both parties and provided that the buyer would pay the contract price of $1,000 upon delivery. On February 3, the buyer and the seller orally agreed to postpone delivery until March 1. However, when the widgets arrived on March 1, the buyer refused to accept or pay for the widgets. If the seller sues the buyer for breach of contract, who is most likely to succeed in the action? A. The buyer, because any modification of the parties' contract must satisfy the statute of frauds. B. The buyer, because the agreement on February 3 was not supported by consdieration. C. The seller, because the contract modification on February 3 was immediately binding on both parties. D. The seller, because the oral agreement on February 3 waived the February 20 delivery date. D.

D. The seller, because the oral agreement on February 3 waived the February 20 delivery date. *Here, the buyer's duty to pay under the original contract was conditioned on the seller's delivery by Feb. 20. However, the buyer waived the original delivery date by orally agreeing on Feb. 3 to postpone deliver to March 1. *The seller detrimentally relied on that waiver. Therefore, the seller will likely succeed in this breach-of-contract action* *ALSO, the contract modification (i.e., the waiver) on Feb. 3 did not become binding until the seller detrimentally relied on it.*

A refrigeration-unit manufacturer contracted with a kitchen appliance store to sell and deliver 100 refrigeration units to the store at a price substantially lower than market value. The written and signed contract included the term "F.O.B. kitchen appliance store, on or before March 30." The shipping company that the manufacturer normally used to deliver its refrigeration units experienced an unforeseen strike at the end of March. As a result, the manufacturer personally delivered the units to the store on April 18. The store suffered no material harm due to the delay. The refrigeration appliance industry generally allows appliance manufacturers a 30-day leeway for any contractually specified time of delivery, unless such leeway is expressly prohibited by the contract. If the store brings suit against the manufacturer for breach of contract, which of the following facts provides the manufacturer with the strongest defense to the store's claim? A. The delay was caused by an unforeseeable strike. B. The manufacturer believed that due to the price at which it offered the refrigeration units, the store would accept a late delivery. C. The store suffered no material harm from the delay. D. There is evidence of a trade usage in the refrigeration appliance industry allowing a 30-day leeway for appliance deliveries.

D. There is evidence of a trade usage in the refrigeration appliance industry allowing a 30-day leeway for appliance deliveries. *Under the UCC, a party may explain or supplement the terms of a written contract with evidence of trade usage—i.e., any practice or method of dealing in the particular business or industry that is observed with such regularity so as to justify an expectation that it will be observed in the instant case.*

A groom left his bride at the altar on the day of their wedding. The bride could not bear to keep any painful reminders of the occasion, so she offered to sell her wedding dress to one of her bridesmaids for $5,000. The bride stated that the offer would remain open for 30 days. The bridesmaid said that she was interested but would have to think about it. A week later, the bridesmaid emailed the bride to ask if the price included the custom-made veil that the bride had worn. The bride did not respond to the bridesmaid's question. Within the 30-day period, the bridesmaid accepted the bride's initial offer of $5,000 for the wedding dress. In response, the bride stated that the bridesmaid could only buy the wedding dress for $6,000. Was a contract formed when the bridesmaid accepted the initial offer of $5,000? A. No, because the bride raised the price of the dress to $6,000. B. No, because the bridesmaid's question acted as a counteroffer and a rejection of the $5,000 offer price. C. Yes, because the bride was required to keep the initial offer open for the 30-day period. D. Yes, because the bridesmaid's question did not constitute a counteroffer.

D. Yes, because the bridesmaid's question did not constitute a counteroffer. *An offer that specifies a date on which it will terminate will automatically terminate on the specified date. But unless consideration was paid to keep this option open, the offer can be terminated at an earlier date—e.g., if the offeree makes a counteroffer.*

A farmer owned a tractor and offered his brother the chance to purchase it. The farmer told the brother that he had to decide whether he wanted to purchase the tractor within "six months of today's date." The brother paid the farmer $200 that day to keep the option open. The agreement was reduced to writing, signed by both men, and dated May 15. The farmer died on July 1. On August 15, the brother notified the executor of the farmer's estate that he wanted to accept the offer to buy the tractor. The executor refused to sell, and the brother filed suit for the enforcement of the contract. Is the brother likely to prevail? A. No, because at the time of the farmer's death, the tractor went to his estate. B. No, because the offer terminated on July 1. C. Yes, because the brother made an enforceable contract to buy the tractor on May 15. D. Yes, because the brother paid $200 to keep the option open.

D. Yes, because the brother paid $200 to keep the option open. *An option contract will not terminate if the offeror prematurely dies or becomes mentally incapacitated.

A homeowner called a septic cleaning company and made arrangements for the company to remove the waste from the septic tank on the homeowner's property. After completing the job, the company mailed the homeowner a bill for $500, the fair market value of the services rendered by the company. The bill indicated that payment was due in 60 days. Upon receiving the bill, the homeowner called the company and informed it that, since he had lost his job due to an accident, he would not be paying the company's bill. The following day, the company filed suit for breach of contract. Ten days later, the homeowner moved to dismiss the suit. The court granted the motion, dismissing the suit without prejudice. Is the court's dismissal proper? A. No, because the parties' dealings created an implied-in-fact contract. B. No, because the homeowner has repudiated the contract. C. Yes, because the company failed to demand assurances. D. Yes, because the company's complaint is premature.

D. Yes, because the company's complaint is premature. *Anticipatory Repudiation: generally applies when a contracting party clearly and unequivocally indicates an unwillingness to perform a promise before the time of performance is due. Under repudiation, nonrepudiating party may: 1. treat the repudiation as a breach of the contract; OR 2. ignore the repudiation and demand performance. *THIS DOES NOT apply if date of performance has passed and the nonrepudiating party has fully performed. They cannot sue then until repudiating party's performance is due. (payment)*

A manufacturer of T-shirts contracted with a brand-new clothing store to sell the store 1,000 T-shirts per month for a period of two years. The clothing store's signature color for its clothing was an orange-tinted red color, called coquelicot, which is very difficult to replicate on a consistent basis. The final, written contract specified that any T-shirts that were not coquelicot could be returned, but it was silent with regard to the return of T-shirts for other reasons. One year into the contract, the store decided to switch to coquelicot-colored baseball caps instead of T-shirts. As a result, the store returned the most recent shipment of coquelicot-colored T-shirts to the manufacturer and demanded a refund. The manufacturer refused to grant the refund, and the store sued the manufacturer for damages. At trial, the manufacturer introduced the contract, which clearly stated that T-shirts that were not coquelicot could be returned. The store then attempted to introduce evidence that it had returned coquelicot-colored T-shirts to the manufacturer over the past year without objection and received a refund. Is this evidence admissible? A. No, because evidence regarding the return of the T shirts violates the parol evidence rule. B. No, because the express term in the contract regarding the return of T-shirts takes precedence over the course of performance. C. Yes, because the evidence can reasonably establish the parties' course of dealing on this issue. D. Yes, because the evidence is relevant to show that the manufacturer had accepted the return of coquelicot-colored T-shirts in the past.

D. Yes, because the evidence is relevant to show that the manufacturer had accepted the return of coquelicot-colored T-shirts in the past. *Under the UCC parol evidence rule, course of performance can be used to supplement or explain the terms of a final written agreement.*

On November 1, the owner of a yacht posted a flyer at a local coffee shop reading, "Yacht for Sale: Make me an offer!" The flyer also included the owner's phone number. A buyer called the owner on November 3 to ask how much the owner wanted for the yacht. The owner said, "Well, I'd hate to part with it for less than $55,000, but if you can pay me $50,000 by November 20, I'd sell it to you. I'll hold onto the yacht for you until then." Elated, the buyer took steps to obtain a loan by November 20. On November 15, a second buyer called the owner and offered to buy the yacht for $60,000. The owner immediately accepted, and the second buyer picked up the yacht the next day. On November 20, having obtained a loan, the first buyer visited the owner with a check for $50,000. The first buyer then learned the owner had already sold the yacht. Can the first buyer bring a successful suit against the owner for breach of contract? A. No, because the owner's statement to the first buyer was only an invitation to deal. B. No, because the second buyer offered more money for the yacht than the first buyer agreed to pay. C. Yes, because the owner promised to keep the offer open for a specific period of time. D. Yes, because the owner's offer to the first buyer was still outstanding on November 20.

D. Yes, because the owner's offer to the first buyer was still outstanding on November 20. *Had the first buyer found out about the second buyer, then there would have been a constructive revocation. Because the first buyer did not find out until AFTER his acceptance (performance) a contract had been formed. *therefore, owner breached contract.*

A nonprofit organization that provides summer camps for at-risk youth hosted a walkathon to raise money to provide stipends for summer-camp expenses to 350 low-income families. The organization solicited various businesses to sponsor the walkathon, including an executive of a large food-services company. The executive wanted the organization to hire his food-services company to provide food services to the organization's summer camps when the organization's contract with their current food-services provider expired in one month. Hoping to catch the organization's attention, the executive pledged that his company would match all of the sponsors' pledges. Because he was a business sponsor, the executive was required to fill out and sign a writing stating this promise. The walkathon was a success, and the total amount raised, excluding the executive's pledge, was $42,000. After collecting the pledge money from all of the sponsors except for the executive, the organization was able to issue stipends to all 350 families. The organization planned to use the executive's pledge to repair some of the older cabins at the summer camp. Prior to being asked for his matching pledge, the executive learned that the contract of the organization's current food-services provider had been renewed for another five years. The executive subsequently repudiated his promise to match all of the sponsors' pledges. If the nonprofit organization sues the executive for $42,000, will it likely succeed? A. No, because the organization did not actually rely to its detriment on the executive's promise to match all of the sponsors' pledges. B. No, because there is no substantial injustice since the organization was able to issue all 350 stipends without the executive's pledge. C. Yes, because the executive acted in bad faith when he repudiated his promise to match the pledges of the other sponsors. D. Yes, because the executive's promise to match the pledges of the other sponsors is enforceable without proof of detrimental reliance or substantial injustice.

D. Yes, because the executive's promise to match the pledges of the other sponsors is enforceable without proof of detrimental reliance or substantial injustice. *A charitable subscription—i.e., a written promise to contribute money or property to a charitable institution—is enforceable on promissory-estoppel grounds without proof of detrimental reliance or substantial injustice. All that is needed is proof that the promisor reasonably expected to induce reliance on the promise.* **When it comes to charitable subscriptions—i.e., written promises to contribute money or property to a charitable institution—most courts find that proof of detrimental reliance is not required.* Therefore, a charitable institution need only demonstrate that the promisor reasonably expected to induce reliance on the promise.**

A general contractor learned that a company was accepting bids for a lucrative construction project involving a high-rise building. The general contractor contacted a number of subcontractors and informed them that he would be accepting bids for the electrical work on the project for the next week. After receiving a number of bids from subcontractors, the general contractor selected a bid from a young subcontractor, which was the lowest bid but still within a reasonable range of the other bids. The general contractor used that sub-bid in calculating his overall bid on the construction project. Soon after submitting his sub-bid to the general contractor and after the general contractor had submitted his overall bid to the company, the young subcontractor realized that he could have charged more for his services based on their market value. The company ended up choosing the general contractor's bid for the project, and later that same day, the young subcontractor told the general contractor that he was revoking his sub-bid for the electrical work. As a result, the general contractor had to use a different subcontractor to perform the work at a cost $3,000 higher than the young subcontractor's bid. In a suit to recover the $3,000 from the young subcontractor, is the general contractor likely to prevail? A. No, because the young subcontractor's sub-bid was an offer that could be freely revoked. B. No, because the young subcontractor's sub-bid was an offer that could be freely revoked. C. Yes, because an enforceable contract was formed when the general contractor used the young subcontractor's sub bid in his overall bid on the project. D. Yes, because the general contractor detrimentally relied on the young subcontractor's sub-bid.

D. Yes, because the general contractor detrimentally relied on the young subcontractor's sub-bid. *Under the doctrine of promissory estoppel (i.e., detrimental reliance), an offer is binding as an option contract and therefore irrevocable for a reasonable period of time if: a. the offeror should have reasonably expected to induce reliance on the offer before acceptance; b. the offeree reasonably relied on the offer through action or forbearance; c. the offeree suffered substantial detriment as a result of such reliance AND d. injustice can be avoided only by enforcing the offer. *while no contract was formed, it was an irrevocable offer that the general contractor detrimentally relied on. General contractor can recover under theory of promissory estoppel.

A homeowner entered into a written agreement with a company to build a swimming pool in the homeowner's backyard for $40,000, to be paid upon completion of the pool. The company delegated its duty to an independent contractor. The independent contractor began the excavation for the pool, but after realizing that the costs would be higher than anticipated, it abandoned the project. The homeowner hired a partnership to complete the pool for $50,000. Can the homeowner sue the company for its expectation damages of $10,000? A. No, because the company was released from liability. B. No, because the high costs rendered performance under the contract impracticable. C. Yes, because the homeowner's lack of consent invalidated the delegation. D. Yes, because the independent contractor did not perform.

D. Yes, because the independent contractor did not perform. *When a promisor delegates a contractual obligation to a nonparty (delegatee), the promisor is not released from liability unless there is a novation. Novation requires: 1. that the promisor repudiate liability; AND 2. that the promisse accept performance from the delegatee without reserving rights against the promisor.*

A nature magazine advertised a photography contest in its January issue, offering "$1,000 to any subscriber who sends us a photograph of the rare Florida Grasshopper Sparrow that we use for the cover of our May issue. Only submissions meeting our technical specifications and received by April 1 will be considered." The only subscriber to respond to the advertised contest sent the magazine a photograph of the sparrow that met the magazine's technical specifications. The photograph arrived on March 15. However, due to an ecological disaster that occurred in early April, the magazine decided to use a different picture on the cover of its May issue. The magazine used the subscriber's picture on the cover of its June issue and has refused to pay $1,000 to the subscriber on the ground that it was not used on the May cover. Is the subscriber likely to prevail in a breach-of-contract action against the nature magazine? A. No, because the subscriber's photograph was not used on the cover of the May issue. B. No, because the subscriber failed to adequately notify the magazine of his acceptance. C. Yes, because all of the express conditions of the offer have been satisfied. D. Yes, because the magazine prevented the publication of the photograph.

D. Yes, because the magazine prevented the publication of the photograph. *A condition precedent to a contractual duty to perform will be excused if a party whose performance is subject to that condition wrongfully prevents th e condition from occurring -- e.g. by breaching the implied duty of good faith and fair dealing. *Ex. Magazine prevented publication of photograph in question.

A private port authority contracted with a company that manufactures and operates cranes to assist with loading and unloading containers from ships docked at the port. One of the company's cranes was defectively manufactured. Due to this defect, a container was dropped, injuring an individual below. The individual sued the port authority, alleging negligence. Neither the individual nor the port authority notified the crane company of this lawsuit. The port authority settled its claim with the individual before trial for a reasonable amount. The port authority seeks to recover the cost of the settlement from the crane company under a breach-of-contract action. Is the port authority likely to prevail? A. No, because damages for personal injury cannot be recovered in a breach-of-contract action. B. No, because the port authority settled the lawsuit rather than litigating the matter to a final judgment. C. Yes, because the crane company is liable for all consequences flowing from its breach of the contract. D. Yes, because the settlement was reasonably foreseeable at the time the contract was formed.

D. Yes, because the settlement was reasonably foreseeable at the time the contract was formed. *Consequential Damages: Damages for losses stemming from nonbreaching party's special circumstances if breaching party: 1) knew of those special circumstances; OR 2) could have reasonably foreseen harm caused by breach. *Consequential damages—i.e., losses arising from the parties' special circumstances—are recoverable only if they were reasonably foreseeable to the breaching party when the contract was entered.*

A party-planning company specialized in creating and selling nine different kits for themed parties. A store that sells party-related items entered into a written agreement with the company. Under this agreement, the company was to deliver 500 kits to the store by November 1. The agreement stated that selections regarding the types of kits and the number of each were to be made by October 15, but the agreement did not specify who was to make the selections. Neither the store nor the company selected any assortment of the kits by October 15. On October 16, the company notified the store that due to its breach, the company would not be shipping the party kits. On October 17, after receiving the company's notification, the store informed the company of its selections. The company refused to send the kits that the store selected even though it had a surplus of all of the merchandise and could have filled the store's order with any combination of themed kits. If the store sues the company for breach of contract on November 2, is the store likely to prevail? A. No, because the company had no duty to perform since an assortment was not selected by October 15. B. No, because the failure to specifiy the party responsible for selecting the types and numbers of each kit renders the contract unenforceable due to the indefiniteness of its terms. C. Yes, because the company was required to make a reasonable selection of available merchandise to fill the order. D. Yes, because the store's two day delay in making its selections did not have a material effect on the company's ability to perform the contract.

D. Yes, because the store's two day delay in making its selections did not have a material effect on the company's ability to perform the contract. *When a contract for the sale of assorted goods does not specify who will choose the assortment, the UCC imposes a duty on the buyer to make that selection. If the buyer fails to specify the assortment of goods, then the seller can treat that failure as a breach -- but only if the buyer's failure to specify the assortment materially impacts the seller's performance.* **the store's failure to specify the assortment did NOT materially impact the company -- therefore, the company breached the contract.**

A theater owner wanted to renovate the interior of the classic theater that he owned. The theater owner contacted a light vendor, and the parties began negotiating a deal for the purchase of unique stage and overhead lighting for the theater. After several phone calls, the light vendor sent a letter to the theater owner that stated: "40 overhead fixtures and 14 stage lights to be delivered on the first of the month." The theater owner, intending to form a contract, responded with a signed letter containing the following: "Delivery, fixtures, and stage lights sound good. Installation would also be helpful." The light vendor received the theater owner's letter. Does a contract that binds the theater owner and the light vendor exist? A. No, because the light vendor is a merchant and the theater owner's acceptance materially altered the original offer. B. No, because the price of the lighting fixtures was not specified in the written offer or acceptance. C. No, because the theater owner's counteroffer acted as a rejection of the original offer. D. Yes, because the theater owner and the light vendor intended to create a contract.

D. Yes, because the theater owner and the light vendor intended to create a contract. *Under the UCC, an acceptance that includes new or revised terms is effective so long as it is not conditioned upon the offeror's agreement to such terms.*

A produce wholesaler sent a written offer to a farmer to purchase all of the corn that the wholesaler required for his business from the farmer for a period of two years. Excited at the prospect of having a guaranteed sale for his corn, the farmer immediately communicated his acceptance to the wholesaler. The wholesaler and the farmer entered into a written contract reflecting the basic terms set forth in the wholesaler's offer. Six months after the contract was executed, the wholesaler determined that, while the farmer's corn was returning a profit, the farmer's corn was not selling as well as corn that the wholesaler could acquire from other sources. The wholesaler contacted the farmer and informed him that he no longer required any of the farmer's corn and would not be placing another order. The wholesaler immediately started buying his corn from another source. If the farmer sues the wholesaler for breach of contract, is he likely to prevail? A. No, because the contract did not contain a specific quantity term. B. No, because the wholesaler no longer needed the farmer's goods. C. Yes, because the farmer relied on the wholesaler's promise. D. Yes, because the wholesaler purchased corn from another source.

D. Yes, because the wholesaler purchased corn from another source. *A requirements contract is a contract for the sale of as many goods as the buyer requires during a specified period. This creates an exclusive agreement between the buyer and the seller.* **The duty of good faith and fair dealing implied in every contract requires the buyer to purchase goods from the seller only. A failure to do so violates that duty and is a breach of contract.**

A woman sent an offer to sell her office printer to her friend for $450. In her offer, the woman said that the friend was welcome to mail her acceptance to the woman's business address but that the friend had to let the woman know within the next week whether she was interested. The friend needed an office printer, so she immediately accepted the woman's offer by mailing a letter to the woman's home address. Later that same week, thinking that the friend was not interested, the woman sold the office printer to a different person. A few days later, after the one-week deadline had passed, the friend's letter was delivered to the woman's house. The woman called the friend thereafter and told her that the office printer had already been sold. Will the friend likely succeed in an action for breach of contract? A. No, because the offeror determines the manner and means by which an offer may be accepted. B. No, because the woman did not receive the friend's acceptance letter until after the one-week deadline had passed. C. Yes, because the offer was irrevocable for at least one week. D. Yes, because the woman did not specify that mailing an acceptance to her business address was the only mode of acceptance.

D. Yes, because the woman did not specify that mailing an acceptance to her business address was the only mode of acceptance. *Here, the woman did not dictate that an acceptance must be mailed to her business address; she merely welcomed the friend to mail it there. Therefore, the friend could accept the offer by another reasonable means. Since that acceptance was effective upon dispatch, the friend accepted the woman's offer before the one-week deadline had passed* **An offeror can dictate the manner and means by which the offer may be accepted. But if the offeror does not do so, then the offeree can accept the offer in any reasonable manner and by any reasonable means—e.g., delivering the acceptance by mail, which is effective upon dispatch.**

A father, hoping to build a new playground for his children, invited a friend whose hobby was woodworking to lunch. During the lunch, the two men discussed an arrangement in which the woodworker would build and deliver a swing set to the father for $2,000 within two weeks. Later that day, the woodworker sent an email to the father restating what had been discussed. The father immediately responded in a signed email stating, "We have a deal. But please deliver the set within one week instead." The woodworker did not respond but began working on the swing set that day. Eight days later, the father called the woodworker to ask why the swing set had not been delivered. The woodworker stated that he intended to deliver the swing set within the two-week period originally discussed. He began to work more quickly to complete the swing set sooner and delivered the swing set two days early, but the father refused to pay him for it. Under the UCC, is the woodworker entitled to recover the $2,000? A. No, because the parties did not agree to all essential terms. B. No, because the woodworker did not deliver the swing set within one week. C. Yes, because the goods were specially manufactured. D. Yes, because the woodworker delivered the swing set within two weeks.

D. Yes, because the woodworker delivered the swing set within two weeks. *Battle-of-the-forms rule: Under this rule, an acceptance that contains new or revised terms is still an acceptance (not a counteroffer) so long as it does not require assent to new or revised terms. And if at least one party is a nonmerchant, then the new or revised terms are merely treated as proposed additions to the contract.* **Under the UCC, an acceptance that contains new or revised terms is still an acceptance so long as assent to the new or revised terms is not required. And if at least one of the contracting parties is a nonmerchant, then the new or revised terms are merely treated as proposed additions to the contract.**

A new florist placed a written order with a wholesaler for $15,000 worth of fresh flowers. Delivery was to be made to the florist's shop via a national delivery service. Because the florist was a new customer, the wholesaler accepted the order on the condition that the florist pay $5,000 in advance and the remaining $10,000 within 20 days of delivery. There was no discussion as to who bore the risk of loss. The florist paid the wholesaler $5,000, and the wholesaler arranged with a national delivery service to pick up and deliver the flowers to the florist. The delivery service picked up the flowers, but, due to malfunction of the temperature controls on the transporting plane, the flowers were worthless upon arrival. The florist rejected the flowers and notified the wholesaler, who refused to ship other flowers. The wholesaler filed a claim against the florist for the remaining $10,000. The florist counterclaimed for the return of his $5,000 payment to the wholesaler. How should the court rule on these claims? A. Deny both claims, because the florist accepted the risk of loss up to the amount he had paid for the goods. B. Grant the florist's claim for $5,000 and deny the wholesaler's claim for $10,000, because the risk of loss remained with the wholesaler. C. Grant the wholesaler's claim for $10,000 and deny the florist's claim for $5,000, because the risk of loss passed to the florist. D. Offset the two claims against each other and require the florist to pay the wholesaler $2,500, because each party should bear the loss equally.

Grant the florist's claim for $5,000 and deny the wholesaler's claim for $10,000, because the risk of loss remained with the wholesaler. *Define Shipment Contract versus Destination Contract. A destination contract forms when a destination is specified in the contract (aka Flower shop). Because destination was specified, the burden of risk did no shift to buyer until flowers reached florist's shop.*

In January, a local farmer contracted with a chef to sell the chef a specified amount of local organic tomatoes to be delivered on August 1. On June 15, the farmer called the chef to tell him that part of his crop was infested with tomato fruitworms and he was unsure that he would be able to deliver the full amount requested by August 1. The chef told the farmer that it was absolutely essential that he receive those tomatoes on time to make organic tomato sauce for a restaurant scheduled to open in late August. The farmer assured the chef that he would do his very best to save the crop and deliver by August 1. Does the chef have valid legal grounds to cancel the contract and order tomatoes from another source? A. No, because the farmer did not state unequivocally that he could not deliver the tomatoes on time. B. No, because the farmer still had more than 30 days to deliver the tomatoes. C. Yes, because the farmer committed an anticipatory repudiation of the contract by causing the chef to feel insecure about the farmer's performance. D. Yes, because the farmer failed to provide adequate assurances to the chef.

No, because the farmer did not state unequivocally that he could not deliver the tomatoes on time. *Anticipatory repudiation occurs when a party clearly and unequivocally communicates that it does not intend to perform. Mere insecurity about the party's ability to perform is not a repudiation, but it allows the insecure party to demand assurance of performance. Failure to provide adequate assurances is a breach.*


Kaugnay na mga set ng pag-aaral

LONG DIVISION PRACTICE PROBLEMS!

View Set

Adult Nursing - Chapter 63: Assessment and Management of Patients With Eye and Vision Disorders - PrepU

View Set

piaget's 4 stages of cognitive development

View Set

9.5 Relativity Certified User - Practice Questions from Quizzes

View Set

Public Health - Chapter 10: Health Insurance and Healthcare System

View Set